Anda di halaman 1dari 89

My Profile

Exam Revision
Work Smart

Scores

Work Hard

Community
Mock Tests

Sign Out

Help
Group Learning 3

Revision Advice

Tags

Learning Journal

Work Smart
Which of the following is not a feature of cannabinoids?

Test Analysis

(Please select 1 option)


9-tetrahydrocannabinol is the active constituent of the resin
Bioavailability after oral administration is about 80%
Inhibits eicosanoid synthesis
Lowers intraocular pressure
Naloxone blocks the antinociceptive actions of cannabinoids

Submit answer

Skip question

Score:
Total Answered:

Question Navigator
1

10

11

12

13

14

15

16

17

18

19

20

21

22

23

24

25

26

27

28

29

30

31

32

33

34

35

36

37

38

39

40

41

42

43

Tags
Tag
Tag

Hide Question

2014 BMJ Publishing Group Ltd . All rights reserved.


Contact . Corporate . Terms and conditions & Privacy policy

My Profile

Exam Revision
Work Smart

Scores

Work Hard

Community
Mock Tests

Sign Out

Help
Group Learning 3

Revision Advice

Tags

Learning Journal

Work Smart
Which of the following is not a feature of cannabinoids?

Answer Statistics

(Please select 1 option)


9-tetrahydrocannabinol is the active constituent of the resin
Bioavailability after oral administration is about 80%

Incorrect answer selected

Test Analysis

This is the correct answer

Inhibits eicosanoid synthesis


Lowers intraocular pressure
Naloxone blocks the antinociceptive actions of cannabinoids

Cannabinoids are derived from the resin of cannabis sativa, and 9-tetrahydrocannabinol (9-THC) is its most
important pharmacologically active constituent.
Oral bioavailability of THC, whether given in the pure form or as THC in marijuana, is low and extremely
variable, ranging between 5% and 20%, with effects occurring 0.5-3 hours later. Bioavailability of THC in a
marijuana cigarette or pipe also rarely exceeds 10 - 20%.
Naloxone and other opioid receptor antagonists block the analgesic actions of cannabinoids.

Score:

Synthetic cannabinoids reduce arachidonic acid-induced inflammation by inhibiting eicosanoid production.

Total Answered:

Feedback
Next question

Go to summary

Rate this question

Related Articles (BMJ)

Leave question feedback

Question Navigator
1

10

11

12

13

14

15

16

17

18

19

20

21

22

23

24

25

26

27

28

29

30

31

32

33

34

35

36

37

38

39

40

41

42

43

Revision Notes
Any revision notes recorded
here will be automatically
added to a list, and can be

My Profile

Exam Revision
Work Smart

Scores

Work Hard

Community
Mock Tests

Sign Out

Help
Group Learning 7

Revision Advice

Tags

Learning Journal

Work Smart
A patient who is listed for excision of his operable squamous cell lung cancer, suffers a life threatening

Test Analysis

haemoptysis on the ward.


Which of the following is the most appropriate treatment?
(Please select 1 option)
Antibiotics
Bronchial embolisation
Conservative care
Radiotherapy
Tranexamic acid

Submit answer

Skip question

Go to summary

Score:
Total Answered:

Question Navigator
1

10

11

12

13

14

15

16

17

18

19

20

21

22

23

24

25

26

27

28

29

30

31

32

33

34

35

36

37

38

39

40

41

42

43

Tags
Tag
Tag

Hide Question

2014 BMJ Publishing Group Ltd . All rights reserved.


Contact . Corporate . Terms and conditions & Privacy policy

My Profile

Exam Revision
Work Smart

Scores

Work Hard

Community
Mock Tests

Sign Out

Help
Group Learning 7

Revision Advice

Tags

Learning Journal

Work Smart
A patient who is listed for excision of his operable squamous cell lung cancer, suffers a life threatening

Answer Statistics

haemoptysis on the ward.


Which of the following is the most appropriate treatment?

Test Analysis

(Please select 1 option)


Antibiotics

Incorrect answer selected

Bronchial embolisation

This is the correct answer

Conservative care
Radiotherapy
Tranexamic acid

Life threatening haemoptysis is a medical emergency that requires prompt action.


Pulmonary angiography will identify the blood supply to the tumour and embolisation of this vessel(s) will
immediately stem the bleeding.
Score:

The other options are also important in the long term management.

Total Answered:
Antibiotics are used to prevent secondary bacterial infection.

Feedback
Next question

Go to summary

Rate this question


Leave question feedback

Related Articles (BMJ)


Question Navigator
1

10

11

12

13

14

15

16

17

18

19

20

21

22

23

24

25

26

27

28

29

30

31

32

33

34

35

36

37

38

39

40

41

42

43

Revision Notes
Any revision notes recorded
here will be automatically
added to a list, and can be

My Profile

Exam Revision
Work Smart

Scores

Work Hard

Community
Mock Tests

Sign Out

Help
Group Learning 7

Revision Advice

Tags

Learning Journal

Work Smart
An 82-year-old lady was admitted to hospital with fever and confusion.

Test Analysis

On examination her temperature was 39C, blood pressure was 80/45 mmHg with a pulse of 110 beats per
minute regular with numerous petechiae over the abdomen.
After taking blood, it was noted that there was continued bleeding from the venous puncture site.
A urinary catheter was inserted and yielded 1500 ml of cloudy yellow offensive-smelling urine.

Which of the following is correct concerning this patient?


(Please select 1 option)
Circulating levels of activated protein C (aPC) will be reduced
Levels of D dimer will be reduced
Levels of fibrin degradation products (FDPs) will be reduced
The activated partial thromboplastin time (APTT) will be below the normal range

Score:

The platelet count is likely to be elevated

Total Answered:

Question Navigator
Submit answer

Skip question

Go to summary

10

11

12

13

14

15

16

17

18

19

20

21

22

23

24

25

26

27

28

29

30

31

32

33

34

35

36

37

38

39

40

41

42

43

Tags
Tag
Tag

Hide Question

2014 BMJ Publishing Group Ltd . All rights reserved.


Contact . Corporate . Terms and conditions & Privacy policy

My Profile

Exam Revision
Work Smart

Scores

Work Hard

Community
Mock Tests

Sign Out

Help
Group Learning 7

Revision Advice

Tags

Learning Journal

Work Smart
An 82-year-old lady was admitted to hospital with fever and confusion.

Answer Statistics

On examination her temperature was 39C, blood pressure was 80/45 mmHg with a pulse of 110 beats per
minute regular with numerous petechiae over the abdomen.

Test Analysis

After taking blood, it was noted that there was continued bleeding from the venous puncture site.
A urinary catheter was inserted and yielded 1500 ml of cloudy yellow offensive-smelling urine.

Which of the following is correct concerning this patient?


(Please select 1 option)
Circulating levels of activated protein C (aPC) will be reduced

Correct

Levels of D dimer will be reduced


Levels of fibrin degradation products (FDPs) will be reduced
The activated partial thromboplastin time (APTT) will be below the normal range
The platelet count is likely to be elevated

Score:
Total Answered:

The patient has disseminated intravascular coagulation (DIC) secondary to sepsis.


Given the history, the most likely source is the urinary tract.

Feedback

Several coagulation abnormalities are seen in sepsis including:

Rate this question


APTT elevated
PT elevated
FDPs elevated
D dimers elevated
Platelets reduced
Fibrinogen reduced
Protein C reduced
Antithrombin reduced.

Next question

Related Articles (BMJ)

Leave question feedback

Question Navigator

Go to summary

10

11

12

13

14

15

16

17

18

19

20

21

22

23

24

25

26

27

28

29

30

31

32

33

34

35

36

37

38

39

40

41

42

43

Revision Notes
Any revision notes recorded
here will be automatically
added to a list, and can be

My Profile

Exam Revision
Work Smart

Scores

Work Hard

Community
Mock Tests

Sign Out

Help
Group Learning 7

Revision Advice

Tags

Learning Journal

Work Smart
A 34-year-old man with a known history of Crohn's disease was admitted to hospital with abdominal pain and

Test Analysis

features of perforation.
He underwent laparotomy and a perforation of the terminal ileum was found with free faecal fluid in the
abdominal cavity. He was transferred to the intensive care unit (ITU).
Together with traditional antimicrobial and supportive ITU therapy, which of the following therapeutic measures
is most likely to improve this patient's outcome?
(Please select 1 option)
High-dose intravenous corticosteroids
Low-dose intravenous corticosteroids
Recombinant anti-endotoxin antibody
Recombinant human antithrombin III
Recombinant human tissue-factor pathway inhibitor

Score:
Total Answered:

Submit answer

Skip question

Go to summary

Question Navigator
1

10

11

12

13

14

15

16

17

18

19

20

21

22

23

24

25

26

27

28

29

30

31

32

33

34

35

36

37

38

39

40

41

42

43

Tags
Tag
Tag

Hide Question

2014 BMJ Publishing Group Ltd . All rights reserved.


Contact . Corporate . Terms and conditions & Privacy policy

My Profile

Exam Revision
Work Smart

Scores

Work Hard

Community
Mock Tests

Sign Out

Help
Group Learning 7

Revision Advice

Tags

Learning Journal

Work Smart
A 34-year-old man with a known history of Crohn's disease was admitted to hospital with abdominal pain and

Answer Statistics

features of perforation.
He underwent laparotomy and a perforation of the terminal ileum was found with free faecal fluid in the
abdominal cavity. He was transferred to the intensive care unit (ITU).

Test Analysis

Together with traditional antimicrobial and supportive ITU therapy, which of the following therapeutic measures
is most likely to improve this patient's outcome?
(Please select 1 option)
High-dose intravenous corticosteroids

Incorrect answer selected

Low-dose intravenous corticosteroids

This is the correct answer

Recombinant anti-endotoxin antibody


Recombinant human antithrombin III
Recombinant human tissue-factor pathway inhibitor

The use of corticosteroids in sepsis remains controversial.

Score:

Meta-analyses of all the trials of high-dose steroids (for example, methylprednisolone 1 g) have confirmed that
there is either no benefit, or even that there is an adverse effect in septic patients. However, more recent

Total Answered:

randomised controlled trials have suggested that there is a benefit in sepsis when lower physiological doses of
steroids are given.
The precise mechanism is not fully understood, although it is well known that septic patients have low levels of
endogenous steroids.
The production of recombinant human anticoagulants has gathered pace in recent years and several products
have been tested. There have been randomised clinical trials of recombinant human antithrombin III (Kyber
Sept trial), activated protein C (PROWESS trial) and tissue-factor pathway inhibitor (OPTIMIST trial). Of these,
only recombinant activated protein C has shown any significant survival benefit at 28 days. However,
subsequent studies have failed to demonstrate a survival benefit, and have shown an increased bleeding risk.
Activated protein C is therefore no longer recommended for the treatment of sepsis.

Next question

Related Articles (BMJ)

Feedback
Rate this question
Leave question feedback

Question Navigator
1

10

11

12

13

14

15

16

17

18

19

20

21

22

23

24

25

26

27

28

29

30

31

32

33

34

35

36

37

38

39

40

41

42

Go to summary

43

Revision Notes
Any revision notes recorded
here will be automatically
added to a list, and can be

My Profile

Exam Revision
Work Smart

Scores

Work Hard

Community
Mock Tests

Sign Out

Help
Group Learning 7

Revision Advice

Tags

Learning Journal

Work Smart
A 56-year-old man is admitted with epigastric pain after drinking heavily.

Test Analysis

He has a temperature of 36.9C, a pulse of 95/min, a blood pressure of 85/60 mmHg, and a respiratory rate of
32/min.
Investigations reveal:
Haemoglobin

12.6 g/dL
10 9 /L

Platelets

169

White cell count

3.9 10 9 /L

(13.0-18.0)
(150-400)
(4-11)

Which of the following is the diagnosis?


(Please select 1 option)
Leaking aortic aneurysm
Multi-organ dysfunction syndrome (MODS)

Score:

Severe sepsis

Total Answered:

Septic shock
Systemic inflammatory response syndrome (SIRS)

Submit answer

Skip question

Question Navigator

Go to summary

10

11

12

13

14

15

16

17

18

19

20

21

22

23

24

25

26

27

28

29

30

31

32

33

34

35

36

37

38

39

40

41

42

43

Tags
Tag
Tag

Hide Question

2014 BMJ Publishing Group Ltd . All rights reserved.


Contact . Corporate . Terms and conditions & Privacy policy

My Profile

Exam Revision
Work Smart

Scores

Work Hard

Community
Mock Tests

Sign Out

Help
Group Learning 7

Revision Advice

Tags

Learning Journal

Work Smart
A 56-year-old man is admitted with epigastric pain after drinking heavily.

Answer Statistics

He has a temperature of 36.9C, a pulse of 95/min, a blood pressure of 85/60 mmHg, and a respiratory rate of
32/min.

Test Analysis

Investigations reveal:
Haemoglobin

12.6 g/dL
10 9 /L

Platelets

169

White cell count

3.9 10 9 /L

(13.0-18.0)
(150-400)
(4-11)

Which of the following is the diagnosis?


(Please select 1 option)
Leaking aortic aneurysm

Incorrect answer selected

Multi-organ dysfunction syndrome (MODS)


Severe sepsis
Septic shock

Score:
Systemic inflammatory response syndrome (SIRS)

This is the correct answer

Total Answered:
This patient has features of pancreatitis. He also has hypotension, and leucopenia. He therefore fulfils the
criteria for systemic inflammatory response syndrome. This is equivalent to sepsis, but occurs in the absence of
infection (e.g. in pancreatitis).

Feedback
Rate this question

SIRS is defined as two or more of the following:


Leave question feedback
Temperature more than 38C or less than 36C.
Heart rate more than 90 beats/min.
Respiratory rate more than 20 breaths/min or PaCO 2 less than 4.3 kPa.

Question Navigator

WBC count 12,000/mm 3 , less than 4000/mm 3 , or more than 10% immature (bands) form.
A leaking aortic aneurysm is still a possibility, however a decreased white cell count would not be expected.
We do not have enough information to diagnose multi-organ dysfunction. There is no evidence of infection to
make a diagnosis of septic shock or severe sepsis. For information, sepsis is defined as the association of
systemic inflammatory responses with evidence of microbial origin. Severe sepsis also has hypoperfusion or
dysfunction of at least one organ system, and septic shock is this plus hypotension refractory to fluid
resuscitation.

10

11

12

13

14

15

16

17

18

19

20

21

22

23

24

25

26

27

28

29

30

31

32

33

34

35

36

37

38

39

40

41

42

43
Next question

Go to summary

Revision Notes
Related Articles (BMJ)

Any revision notes recorded


here will be automatically
added to a list, and can be

My Profile

Exam Revision
Work Smart

Scores

Work Hard

Community
Mock Tests

Sign Out

Help
Group Learning 7

Revision Advice

Tags

Learning Journal

Work Smart
A 56-year-old man diagnosed with systemic inflammatory response syndrome (SIRS) secondary to pancreatitis

Test Analysis

is admitted to the High Dependency Unit. He has a pulse of 109 beats/min and a blood pressure of 89/74
mmHg despite receiving IV fluids and urine output of 25 ml/hour after catheterisation.
Which of the following is the most appropriate course of action for this patient?
(Please select 1 option)
A central line
A CT abdomen
A surgical referral
An arterial line
Broad spectrum antibiotics

Submit answer

Skip question

Go to summary

Score:
Total Answered:

Question Navigator
1

10

11

12

13

14

15

16

17

18

19

20

21

22

23

24

25

26

27

28

29

30

31

32

33

34

35

36

37

38

39

40

41

42

43

Tags
Tag
Tag

Hide Question

2014 BMJ Publishing Group Ltd . All rights reserved.


Contact . Corporate . Terms and conditions & Privacy policy

My Profile

Exam Revision
Work Smart

Scores

Work Hard

Community
Mock Tests

Sign Out

Help
Group Learning 7

Revision Advice

Tags

Learning Journal

Work Smart
A 56-year-old man diagnosed with systemic inflammatory response syndrome (SIRS) secondary to pancreatitis

Answer Statistics

is admitted to the High Dependency Unit. He has a pulse of 109 beats/min and a blood pressure of 89/74
mmHg despite receiving IV fluids and urine output of 25 ml/hour after catheterisation.

Test Analysis

Which of the following is the most appropriate course of action for this patient?
(Please select 1 option)
A central line

Correct

A CT abdomen
A surgical referral
An arterial line
Broad spectrum antibiotics

Early goal-directed therapy (EGDT) in cases of SIRS or septic shock is becoming increasingly recognised as
potentially beneficial. If fluids are not achieving haemodynamic stability, and there is indication of hypoperfusion
as indicated by oliguria or lactataemia, then vigorous resuscitation is indicated.
Score:
EGDT aims to increase organ perfusion through restoration of mean arterial pressure using inotropes if
necessary, maintaining central venous pressure (CVP), maintaining oxygenation, and using SjVO2 (jugular

Total Answered:

venous oxygen saturation) as a guide to oxygen utilisation at the tissue level. SjVO2 higher than 70% is
indicative of organ hypoperfusion, as oxygen is not being extracted.

Feedback

Insertion of a central line above allows measurement of CVP, SjVO2 and the use of inotropes.

Rate this question


Leave question feedback

Next question

Go to summary

Question Navigator
Related Articles (BMJ)

10

11

12

13

14

15

16

17

18

19

20

21

22

23

24

25

26

27

28

29

30

31

32

33

34

35

36

37

38

39

40

41

42

43

Revision Notes
Any revision notes recorded
here will be automatically
added to a list, and can be

My Profile

Exam Revision
Work Smart

Scores

Work Hard

Community
Mock Tests

Sign Out

Help
Group Learning 7

Revision Advice

Tags

Learning Journal

Work Smart
A 56-year-old man with septic shock is fully ventilated, on continuous veno-venous haemofiltration receiving

Test Analysis

noradrenaline, vancomycin and ciprofloxacin.


He has a mean arterial pressure (MAP) of 60 mmHg which is then not improved after changing from
noradrenaline to adrenaline. There is no evidence of myocardial dysfunction.
Which of the following would be the most appropriate next step in managing this patient?
(Please select 1 option)
ACTH stimulation test
Activated protein C
Change of inotropes
Hydrocortisone
Nitric oxide

Score:
Submit answer

Skip question

Go to summary

Total Answered:

Question Navigator
1

10

11

12

13

14

15

16

17

18

19

20

21

22

23

24

25

26

27

28

29

30

31

32

33

34

35

36

37

38

39

40

41

42

43

Tags
Tag
Tag

Hide Question

2014 BMJ Publishing Group Ltd . All rights reserved.


Contact . Corporate . Terms and conditions & Privacy policy

My Profile

Exam Revision
Work Smart

Scores

Work Hard

Community
Mock Tests

Sign Out

Help
Group Learning 7

Revision Advice

Tags

Learning Journal

Work Smart
A 56-year-old man with septic shock is fully ventilated, on continuous veno-venous haemofiltration receiving

Answer Statistics

noradrenaline, vancomycin and ciprofloxacin.


He has a mean arterial pressure (MAP) of 60 mmHg which is then not improved after changing from
noradrenaline to adrenaline. There is no evidence of myocardial dysfunction.

Test Analysis

Which of the following would be the most appropriate next step in managing this patient?
(Please select 1 option)
ACTH stimulation test

Incorrect answer selected

Activated protein C
Change of inotropes
Hydrocortisone

This is the correct answer

Nitric oxide

The Surviving Sepsis Campaign (a partnership of the Society of Critical Care Medicine, the European Society
of Intensive Care Medicine, and the International Sepsis Forum) has teamed up with the Institute for
Healthcare Improvement to develop severe sepsis bundles. A 'bundle' is a group of interventions related to a
disease process that, when executed together, result in better outcomes than when implemented individually.
Sepsis Resuscitation Bundle:

Score:
Total Answered:

Feedback

Should begin immediately, but must be accomplished within the first six hours of presentation.

Rate this question


1. Serum lactate measured.
2. Blood cultures obtained prior to antibiotic administration.
3. From the time of presentation, broad-spectrum antibiotics administered within three hours for ED
admissions and one hour for non-ED ICU admissions.
4. In the event of hypotension and/or lactate > 4 mmol/l (36 mg/dl):
Deliver an initial minimum of 30 ml/kg of crystalloid (or colloid equivalent).
Apply vasopressors for hypotension not responding to initial fluid resuscitation to maintain mean arterial
pressure (MAP) > 65 mm Hg.
5. In the event of persistent hypotension despite fluid resuscitation (septic shock) and/or lactate > 4 mmol/l
(36 mg/dl):
Achieve central venous pressure (CVP) of > 8 mm Hg.
Achieve central venous oxygen saturation (ScvO2) of > 70%.

Leave question feedback

Question Navigator
1

10

11

12

13

14

15

16

17

18

19

20

21

22

23

24

25

26

27

28

29

30

31

32

33

34

35

36

37

38

39

40

41

42

Sepsis Management Bundle:


To be accomplished as soon as possible may be completed within twenty-four hours of presentation.
1. Steroids administered for septic shock in accordance with a standardised ICU policy. ACTH stimulation test
not required prior to this.
2. Glucose control maintained > lower limit of normal, but < 150 mg/dl (8.3 mmol/l).
3. Inspiratory plateau pressures maintained < 30 cm H2O for mechanically ventilated patients.
Drotrectogin alpha (activated protein C) used to be recommended by NICE for the treatment of severe sepsis.

43

Revision Notes
Any revision notes recorded
here will be automatically
added to a list, and can be

However, in October 2011 the company withdrew this from the market following the results of the PROWESSSHOCK study, which showed there was no statistically significant reduction in 28-day all-cause mortality in

viewed in full on the summary

patients with septic shock.

page.

Nitric oxide is a non-proven therapy in adult respiratory distress syndrome (ARDS), but is less likely to be
effective in this situation.
Reference:
Dellinger RP, Levy MM, Rhodes A, et al. Surviving sepsis campaign: international guidelines for management
of severe sepsis and septic shock: 2012. Crit Care Med. 2013;41(2):580-637.

Next question

Go to summary

Save

Tags
Related Articles (BMJ)
Tag
Tag

Hide Question

2014 BMJ Publishing Group Ltd . All rights reserved.


Contact . Corporate . Terms and conditions & Privacy policy

My Profile

Exam Revision
Work Smart

Scores

Work Hard

Community
Mock Tests

Sign Out

Help
Group Learning 7

Revision Advice

Tags

Learning Journal

Work Smart
A 67-year-old woman is admitted with blackouts.

Test Analysis

Her electrocardiogram shows ventricular escape with complete heart block. As you are standing there she
blacks out once more. Her rhythm strip shows P wave asystole.
Which of the following would be the initial immediate treatment here after airway and breathing?
(Please select 1 option)
Adenosine 6 mg
Adrenaline 1 mg
Atropine 0.6 mg
Transcutaneous pacing
Transvenous pacing

Score:
Submit answer

Skip question

Go to summary

Total Answered:

Question Navigator
1

10

11

12

13

14

15

16

17

18

19

20

21

22

23

24

25

26

27

28

29

30

31

32

33

34

35

36

37

38

39

40

41

42

43

Tags
Tag
Tag

Hide Question

2014 BMJ Publishing Group Ltd . All rights reserved.


Contact . Corporate . Terms and conditions & Privacy policy

My Profile

Exam Revision
Work Smart

Scores

Work Hard

Community
Mock Tests

Sign Out

Help
Group Learning 7

Revision Advice

Tags

Learning Journal

Work Smart
A 67-year-old woman is admitted with blackouts.

Answer Statistics

Her electrocardiogram shows ventricular escape with complete heart block. As you are standing there she
blacks out once more. Her rhythm strip shows P wave asystole.

Test Analysis

Which of the following would be the initial immediate treatment here after airway and breathing?
(Please select 1 option)
Adenosine 6 mg

Incorrect answer selected

Adrenaline 1 mg
Atropine 0.6 mg
Transcutaneous pacing

This is the correct answer

Transvenous pacing

Occasionally, atrial electrical activity continues in the absence of ventricular impulses. This is referred to as Pwave asystole and may respond to electrical pacing. This can be achieved by transvenous, transcutaneous or
manual techniques. Transvenous pacing takes longer to instigate, and transcutaneous pacing is therefore the
initial choice here. Manual pacing is an effective holding measure before more definitive pacing is instituted.

Score:
Total Answered:

Atropine can also be used, as can adrenaline, for sustained P wave asystole but pacing is the initial treatment
of choice. All crash trollies within UK hospitals contain the equipment required to externally pace patients.

Feedback

Adenosine can induce asystole, and is only indicated in the treatment of supraventricular tachycardias.

Rate this question


Next question

Go to summary

Leave question feedback

Question Navigator
Related Articles (BMJ)

10

11

12

13

14

15

16

17

18

19

20

21

22

23

24

25

26

27

28

29

30

31

32

33

34

35

36

37

38

39

40

41

42

43

Revision Notes
Any revision notes recorded
here will be automatically
added to a list, and can be

My Profile

Exam Revision
Work Smart

Scores

Work Hard

Community
Mock Tests

Sign Out

Help
Group Learning 7

Revision Advice

Tags

Learning Journal

Work Smart
A 24-year-old female is admitted with palpitations. Her pulse is 220 beats/min, blood pressure 70/50 mmHg

Test Analysis

and she has a respiratory rate 32/min.


She is awake, alert and oriented but dyspnoeic. Her electrocardiogram shows a regular rhythm with QRS
complex width of 0.11s.
What is the most appropriate therapy for this patient?
(Please select 1 option)
Adenosine 6 mg/6 mg/12 mg
Amiodarone 300 mg
Atenolol 50 mg
Direct current cardioversion
Verapamil 10 mg

Score:
Submit answer

Skip question

Go to summary

Total Answered:

Question Navigator
1

10

11

12

13

14

15

16

17

18

19

20

21

22

23

24

25

26

27

28

29

30

31

32

33

34

35

36

37

38

39

40

41

42

43

Tags
Tag
Tag

Hide Question

2014 BMJ Publishing Group Ltd . All rights reserved.


Contact . Corporate . Terms and conditions & Privacy policy

My Profile

Exam Revision
Work Smart

Scores

Work Hard

Community
Mock Tests

Sign Out

Help
Group Learning 7

Revision Advice

Tags

Learning Journal

Work Smart
A 24-year-old female is admitted with palpitations. Her pulse is 220 beats/min, blood pressure 70/50 mmHg

Answer Statistics

and she has a respiratory rate 32/min.


She is awake, alert and oriented but dyspnoeic. Her electrocardiogram shows a regular rhythm with QRS
complex width of 0.11s.

Test Analysis

What is the most appropriate therapy for this patient?


(Please select 1 option)
Adenosine 6 mg/6 mg/12 mg

Incorrect answer selected

Amiodarone 300 mg
Atenolol 50 mg
Direct current cardioversion

This is the correct answer

Verapamil 10 mg

This is highly likely to be a narrow complex tachycardia.


Strictly speaking, as this patient is showing signs of haemodynamic decompromise (that is, systolic blood
pressure less than 90) she should be immediately DC cardioverted under sedation/anaesthesia.

Score:
Total Answered:

In practice, most people would try adenosine first whilst organising a cardioversion.

Feedback
Next question

Go to summary

Rate this question


Leave question feedback

Related Articles (BMJ)

Question Navigator
1

10

11

12

13

14

15

16

17

18

19

20

21

22

23

24

25

26

27

28

29

30

31

32

33

34

35

36

37

38

39

40

41

42

43

Revision Notes
Any revision notes recorded
here will be automatically
added to a list, and can be

My Profile

Exam Revision
Work Smart

Scores

Work Hard

Community
Mock Tests

Sign Out

Help
Group Learning 7

Revision Advice

Tags

Learning Journal

Work Smart
A 64-year-old man is admitted with central epigastric pain. Abdominal x ray shows a dilated bowel loop.

Test Analysis

His temperature is 37.0C, pulse 130 bpm, blood pressure 80/50 mmHg, respiratory rate 29/min, SpO 2 90% on
air.
His full blood count reveals:
Haemoglobin

130 g/L

(130-180)

White cell count

3.2 10 9 /L

(4-11)

Platelets

108 10 9 /L

(150-400)

MCV

105 fl

(80-96)

Which of the following is the most appropriate initial treatment of this patient?
(Please select 1 option)
100% oxygen

Score:
2 14 gauge venflons and 2 litres Hartmann's

Total Answered:
Intensive care
Intubation and ventilation

Question Navigator

Invasive monitoring

Submit answer

Skip question

Go to summary

10

11

12

13

14

15

16

17

18

19

20

21

22

23

24

25

26

27

28

29

30

31

32

33

34

35

36

37

38

39

40

41

42

43

Tags
Tag
Tag

Hide Question

2014 BMJ Publishing Group Ltd . All rights reserved.


Contact . Corporate . Terms and conditions & Privacy policy

My Profile

Exam Revision
Work Smart

Scores

Work Hard

Community
Mock Tests

Sign Out

Help
Group Learning 7

Revision Advice

Tags

Learning Journal

Work Smart
A 64-year-old man is admitted with central epigastric pain. Abdominal x ray shows a dilated bowel loop.

Answer Statistics

His temperature is 37.0C, pulse 130 bpm, blood pressure 80/50 mmHg, respiratory rate 29/min, SpO 2 90% on
air.

Test Analysis

His full blood count reveals:


Haemoglobin

130 g/L

(130-180)

White cell count

3.2 10 9 /L

(4-11)

Platelets

108 10 9 /L

(150-400)

MCV

105 fl

(80-96)

Which of the following is the most appropriate initial treatment of this patient?
(Please select 1 option)
100% oxygen

Correct

2 14 gauge venflons and 2 litres Hartmann's


Intensive care

Score:

Intubation and ventilation

Total Answered:

Invasive monitoring

Feedback
This patient has systemic inflammatory response syndrome, possibly caused by acute pancreatitis given the
finding of an isolated dilated loop of bowel on adbominal radiograph. However, the initial treatment is the same
independent of the underlying cause.

Rate this question


Leave question feedback

All of the above answers are reasonable, however resuscitation of the sick patient still follows the ABC
algorithm:
Airway
Breathing
Circulation
Airway control and oxygen to maintain normal saturations is the first part of that algorithm. Subsequent fluid
resuscitation and treatment of the underlying cause can then be initiated. The need for invasive monitoring and
intensive care is then assessed, depending on the response to initial treatment.

Next question

Go to summary

Question Navigator
1

10

11

12

13

14

15

16

17

18

19

20

21

22

23

24

25

26

27

28

29

30

31

32

33

34

35

36

37

38

39

40

41

42

43

Related Articles (BMJ)

Revision Notes
Any revision notes recorded
here will be automatically
added to a list, and can be

My Profile

Exam Revision
Work Smart

Scores

Work Hard

Community
Mock Tests

Sign Out

Help
Group Learning 7

Revision Advice

Tags

Learning Journal

Work Smart
A 64-year-old man is admitted with central epigastric pain.

Test Analysis

Abdominal x ray shows a central dilated bowel loop. His temperature is 37.0C, pulse 130 beats per min, blood
pressure 80/50 mmHg, respiratory rate 29/min and SpO 2 90 on air.
His full blood count reveals:
Haemoglobin

13.0 g/dL

(13.0-18.0)

White cell count

3.2 10 9 /L

(4-11)

Platelets

108 10 9 /L

(150-400)

MCV

105 fL

(80-96)

Which of the following is the most likely diagnosis?


(Please select 1 option)
Gall stone ileus
Ischaemic bowel

Score:

Pancreatitis

Total Answered:

Perforated duodenal ulcer

Question Navigator

Small bowel obstruction

Submit answer

Skip question

Go to summary

10

11

12

13

14

15

16

17

18

19

20

21

22

23

24

25

26

27

28

29

30

31

32

33

34

35

36

37

38

39

40

41

42

43

Tags
Tag
Tag

Hide Question

2014 BMJ Publishing Group Ltd . All rights reserved.


Contact . Corporate . Terms and conditions & Privacy policy

My Profile

Exam Revision
Work Smart

Scores

Work Hard

Community
Mock Tests

Sign Out

Help
Group Learning 7

Revision Advice

Tags

Learning Journal

Work Smart
A 64-year-old man is admitted with central epigastric pain.

Answer Statistics

Abdominal x ray shows a central dilated bowel loop. His temperature is 37.0C, pulse 130 beats per min, blood
pressure 80/50 mmHg, respiratory rate 29/min and SpO 2 90 on air.

Test Analysis

His full blood count reveals:


Haemoglobin

13.0 g/dL

(13.0-18.0)

White cell count

3.2 10 9 /L

(4-11)

Platelets

108 10 9 /L

(150-400)

MCV

105 fL

(80-96)

Which of the following is the most likely diagnosis?


(Please select 1 option)
Gall stone ileus

Incorrect answer selected

Ischaemic bowel
Pancreatitis

This is the correct answer

Score:

Perforated duodenal ulcer

Total Answered:

Small bowel obstruction

Feedback
Ischaemic bowel and perforated duodenal ulcer would be high in the differential list. However the history and
raised mean corpuscular volume (MCV) suggests alcohol use and the severity of his observations would
suggest a systemic inflammatory response which is more common with pancreatitis.
Acute pancreatitis has a mortality of 7-10%, often due to sepsis or multi-organ failure. There are a number of
scoring systems which can be used to guide prognosis, but they are unreliable within the first 48 hours o f the
illness. Gallstones account for 50% of cases, with the majority of the rest being associated with alcohol.
Patients typically present with severe epigastric pain which radiates to the back and vomiting. As seen in this
example, there is often a systemic inflammatory response. Amylase is markedly raised, often in excess of four
times the normal value. Early complications include ARDS (adult respiratory distress syndrome), acute kidney
injury and disseminated intravascular coagulation (DIC).
Treatment is essentially supportive, and high levels of monitoring are usually required (often in the intensive
care unit). Those patients who are found to have gallstones should be considered for emergency ERCP, and
all should have a cholecystectomy during the same admission.

Rate this question


Leave question feedback

Question Navigator
1

10

11

12

13

14

15

16

17

18

19

20

21

22

23

24

25

26

27

28

29

30

31

32

33

34

35

36

37

38

39

40

41

42

43
Next question

Go to summary

Revision Notes
Related Articles (BMJ)

Any revision notes recorded


here will be automatically
added to a list, and can be

My Profile

Exam Revision
Work Smart

Scores

Work Hard

Community
Mock Tests

Sign Out

Help
Group Learning 7

Revision Advice

Tags

Learning Journal

Work Smart
A 74-year-old man with ischaemic heart disease is in the surgical high dependency unit following a Hartmann's

Test Analysis

procedure completed 12 hours previously..


He is in pain. He has drained 100/200/300 mls of blood into his drains in each of the last three hours. His blood
pressure is 110/80 mmHg, his pulse 105/min. He has a respiratory rate 32/min, SpO 2 100% on oxygen by face
mask.
Crossmatched blood is available.
Haemoglobin

8.1 g/dL

(13.0-18.0)

White cell count

4.5 10 9 /L

(4-11)

Platelets

132 10 9 /L

(150-400)

Which of the following is the next most appropriate intervention?


(Please select 1 option)
2 units of packed red blood cells

Score:

1000 cmls of crystalloid stat

Total Answered:

1000 ml of colloid stat

Question Navigator

Morphine 5 cmg intravenously

10

11

12

13

14

15

16

17

18

19

20

21

22

23

24

25

26

27

28

29

30

31

32

33

34

35

36

37

38

39

40

41

42

Return to theatre

Submit answer

Skip question

Go to summary

43

Tags
Tag
Tag

Hide Question

2014 BMJ Publishing Group Ltd . All rights reserved.


Contact . Corporate . Terms and conditions & Privacy policy

My Profile

Exam Revision
Work Smart

Scores

Work Hard

Community
Mock Tests

Sign Out

Help
Group Learning 7

Revision Advice

Tags

Learning Journal

Work Smart
A 74-year-old man with ischaemic heart disease is in the surgical high dependency unit following a Hartmann's

Answer Statistics

procedure completed 12 hours previously..


He is in pain. He has drained 100/200/300 mls of blood into his drains in each of the last three hours. His blood
pressure is 110/80 mmHg, his pulse 105/min. He has a respiratory rate 32/min, SpO 2 100% on oxygen by face

Test Analysis

mask.
Crossmatched blood is available.
Haemoglobin

8.1 g/dL

(13.0-18.0)

White cell count

4.5 10 9 /L

(4-11)

Platelets

132 10 9 /L

(150-400)

Which of the following is the next most appropriate intervention?


(Please select 1 option)
2 units of packed red blood cells

Correct

1000 cmls of crystalloid stat


1000 ml of colloid stat

Score:

Morphine 5 cmg intravenously

Total Answered:

Return to theatre

Feedback
He is still actively bleeding and haemodilution will not have been achieved so his Hb will be lower than the
result given. It will take at least 30-60 minutes before he is back in theatre and anaesthetized (CEPOD
lists/emergency sections/crash calls/no porter etc). With IHD he is at at high risk of a perioperative MI. He
should have the blood first as crossmatched blood is available.
His observations could be caused by pain. However in the face of a falling haemoglobin, a rising pulse rate and
an increasing loss into his drains this patient needs to return to theatre.
His blood pressure is not critically low and although many would give crystalloid or colloid it is reasonable in an
elderly man with ischaemic heart disease to give blood if it is easily available. Overload with non-oxygen
carrying fluid should be avoided (although the red cells are not functional for a while).

Next question

Go to summary

Rate this question


Leave question feedback

Question Navigator
1

10

11

12

13

14

15

16

17

18

19

20

21

22

23

24

25

26

27

28

29

30

31

32

33

34

35

36

37

38

39

40

41

42

43

Related Articles (BMJ)


Revision Notes
Any revision notes recorded
here will be automatically
added to a list, and can be

My Profile

Exam Revision
Work Smart

Scores

Work Hard

Community
Mock Tests

Sign Out

Help
Group Learning 7

Revision Advice

Tags

Learning Journal

Work Smart
You are asked to see a 64-year-old man post-oversew of a duodenal ulcer. He is confused.

Test Analysis

His SpO 2 is 97 on oxygen. Pulse 110 beats per min, blood pressure 100/50 mmHg, respiratory rate 32/min
and his urine output is 10 ml in the last hour.
Which of the following is the most appropriate treatment for this man?
(Please select 1 option)
100% oxygen via face mask
Central line and arterial line
Normal saline 500 ml stat
Haloperidol 2.5 mg intravenously
Noradrenaline via central line

Score:
Submit answer

Skip question

Go to summary

Total Answered:

Question Navigator
1

10

11

12

13

14

15

16

17

18

19

20

21

22

23

24

25

26

27

28

29

30

31

32

33

34

35

36

37

38

39

40

41

42

43

Tags
Tag
Tag

Hide Question

2014 BMJ Publishing Group Ltd . All rights reserved.


Contact . Corporate . Terms and conditions & Privacy policy

My Profile

Exam Revision
Work Smart

Scores

Community

Work Hard

Mock Tests

Sign Out

Help
Group Learning 7

Revision Advice

Tags

Learning Journal

Work Smart
You are asked to see a 64-year-old man post-oversew of a duodenal ulcer. He is confused.

Answer Statistics

His SpO 2 is 97 on oxygen. Pulse 110 beats per min, blood pressure 100/50 mmHg, respiratory rate 32/min
and his urine output is 10 ml in the last hour.

Test Analysis

Which of the following is the most appropriate treatment for this man?
(Please select 1 option)
100% oxygen via face mask

Incorrect answer selected

Central line and arterial line


Normal saline 500 ml stat

This is the correct answer

Haloperidol 2.5 mg intravenously


Noradrenaline via central line

Postoperative confusion is common in the elderly however this can be caused by a low perfusion state.
His observations are indicative of underfilling/dehydration. (He could also have atelectais or have developed a
chest infection. Post-operative problems are often multifactorial.)

Score:
Total Answered:

Of the options given a fluid bolus is appropriate. This is what you would expect your FY1 to do while waiting for
the blood test results and before calling you.

Feedback
Some would argue that 100% oxygen comes before fluids but with an SpO 2 of 97 it is unlikely that hypoxia is
contributing to his problems. (nonetheless oxygen therapy would be recommended).

Rate this question

The other options would come further down the line if initial interventions were unsuccessful and he
deteriorated.

Leave question feedback

Question Navigator
Next question

Related Articles (BMJ)

Go to summary

10

11

12

13

14

15

16

17

18

19

20

21

22

23

24

25

26

27

28

29

30

31

32

33

34

35

36

37

38

39

40

41

42

43

Revision Notes
Any revision notes recorded
here will be automatically
added to a list, and can be

My Profile

Exam Revision
Work Smart

Scores

Work Hard

Community
Mock Tests

Sign Out

Help
Group Learning 7

Revision Advice

Tags

Learning Journal

Work Smart
A 67-year-old man is three days post-operation for a sigmoid colectomy. He has insulin dependent diabetes

Test Analysis

mellitus. He complains of dizziness and faintness.


His blood pressure is 80/50 mmHg, his pulse is 110 bpm, he has a respiratory rate 24/min, and he has SpO 2
99% on air. His blood glucose is 18 mmol/L (3.0-6.0 fasting).
His electrocardiogram shows ST depression of 2 mm in leads II, III and AVF.
Which of the following is the initial drug therapy for this patient?
(Please select 1 option)
Aspirin 300mg
Clexane 1 mg/kg subcutaneously
Clopidogrel 75 mg
Diamorphine 2.5 mg

Score:

Glycerol tri-nitrate 800 mcg sublingually

Total Answered:
Submit answer

Skip question

Go to summary

Question Navigator
1

10

11

12

13

14

15

16

17

18

19

20

21

22

23

24

25

26

27

28

29

30

31

32

33

34

35

36

37

38

39

40

41

42

43

Tags
Tag
Tag

Hide Question

2014 BMJ Publishing Group Ltd . All rights reserved.


Contact . Corporate . Terms and conditions & Privacy policy

My Profile

Exam Revision
Work Smart

Scores

Work Hard

Community
Mock Tests

Sign Out

Help
Group Learning 7

Revision Advice

Tags

Learning Journal

Work Smart
A 67-year-old man is three days post-operation for a sigmoid colectomy. He has insulin dependent diabetes

Answer Statistics

mellitus. He complains of dizziness and faintness.


His blood pressure is 80/50 mmHg, his pulse is 110 bpm, he has a respiratory rate 24/min, and he has SpO 2
99% on air. His blood glucose is 18 mmol/L (3.0-6.0 fasting).

Test Analysis

His electrocardiogram shows ST depression of 2 mm in leads II, III and AVF.


Which of the following is the initial drug therapy for this patient?
(Please select 1 option)
Aspirin 300mg

Correct

Clexane 1 mg/kg subcutaneously


Clopidogrel 75 mg
Diamorphine 2.5 mg
Glycerol tri-nitrate 800 mcg sublingually

Initial treatment of an acute coronary syndrome is aspirin 300 mg. This should be safe in the post-surgical
patient with no signs of bleeding at three days post operation. Clexane would also be given, but aspirin initially.

Score:
Total Answered:

The dose of clopidogrel is 300 mg in an acute coronary syndrome.

Feedback
Diamorphine is used to treat anxiety and pain, neither of which is commented upon.

Rate this question

GTN would be reasonable to try, however the blood pressure is low.


Remember that in the diabetic chest pain may not be a feature of acute coronary syndrome due to autonomic

Leave question feedback

dysfunction, and in most post-surgical patients myocardial infarct is silent.

Question Navigator
Next question

Related Articles (BMJ)

Go to summary

10

11

12

13

14

15

16

17

18

19

20

21

22

23

24

25

26

27

28

29

30

31

32

33

34

35

36

37

38

39

40

41

42

43

Revision Notes
Any revision notes recorded
here will be automatically
added to a list, and can be

My Profile

Exam Revision
Work Smart

Scores

Work Hard

Community
Mock Tests

Sign Out

Help
Group Learning 7

Revision Advice

Tags

Learning Journal

Work Smart
A 56-year-old man with severe brain damage is apnoeic, unsedated, and temperature 36.9C.

Test Analysis

He is intubated and ventilated. His biochemistry is normal.


The combination of which of the following specialists would be able to confirm brain stem death?
(Please select 1 option)
Consultant/specialist trainee with one years experience
Consultant/specialist trainee with three years experience
Consultant/specialist trainee with four years experience
Consultant/specialist trainee with five years experience
Consultant/specialist trainee with two years experience

Submit answer

Skip question

Go to summary

Score:
Total Answered:

Question Navigator
1

10

11

12

13

14

15

16

17

18

19

20

21

22

23

24

25

26

27

28

29

30

31

32

33

34

35

36

37

38

39

40

41

42

43

Tags
Tag
Tag

Hide Question

2014 BMJ Publishing Group Ltd . All rights reserved.


Contact . Corporate . Terms and conditions & Privacy policy

My Profile

Exam Revision
Work Smart

Scores

Work Hard

Community
Mock Tests

Sign Out

Help
Group Learning 7

Revision Advice

Tags

Learning Journal

Work Smart
A 56-year-old man with severe brain damage is apnoeic, unsedated, and temperature 36.9C.

Answer Statistics

He is intubated and ventilated. His biochemistry is normal.


The combination of which of the following specialists would be able to confirm brain stem death?

Test Analysis

(Please select 1 option)


Consultant/specialist trainee with one years experience

Incorrect answer selected

Consultant/specialist trainee with three years experience


Consultant/specialist trainee with four years experience
Consultant/specialist trainee with five years experience

This is the correct answer

Consultant/specialist trainee with two years experience

The conventional criteria previously established for clinical death were based upon lack of cardiorespiratory
function.
The development of organ transplantation highlighted patients who had conditions incompatible with life, but
who continued to have some form of cardiorespiratory function with artifical support. This led to a code of
practice for the diagnosis of brainstem death.

Score:

This is based on the knowledge that when the brainstem is damaged to such a degree that its functions are
irreversibly destroyed, the heart will inevitably stop beating shortly afterwards. When this occurs, therefore, the
patient is dead even though respiration and circulation can be artificially maintained. Brain stem function is

Feedback

Total Answered:

checked through set criteria, and the findings must be agreed by at least two senior doctors. One should be a
consultant, and the other must have at least five years post registration who has experience in the testing of

Rate this question

brain stem death.

Leave question feedback

'Life-support' should be withdrawn at this point, but consideration should be taken as to whether the person
would be a suitable organ donor.

Question Navigator

Reference & Further Reading:


A code of practice for the diagnosis of brain stem death, including guidelines for the identification and
management of potential organ and tissue donors. Department of Health. March 1998.

Next question

Related Articles (BMJ)

Go to summary

10

11

12

13

14

15

16

17

18

19

20

21

22

23

24

25

26

27

28

29

30

31

32

33

34

35

36

37

38

39

40

41

42

43

Revision Notes
Any revision notes recorded
here will be automatically
added to a list, and can be

My Profile

Exam Revision
Work Smart

Scores

Work Hard

Community
Mock Tests

Sign Out

Help
Group Learning 7

Revision Advice

Tags

Learning Journal

Work Smart
A 66-year-old male in intensive care has received 2000 mls of colloid in three hours.

Test Analysis

He is receiving an escalating noradrenaline infusion (currently at 0.76 mcg/kg/min), has a blood pressure of
90/50 mmHg, pulse of 90 beats per min, a central venous pressure of 10 mmHg and capillary refill time of 2-3
seconds. His plasma lactate concentration is 2.9 mmol/L (<1.5).
Which of the following is an appropriate method of measuring adequate intravascular filling?
(Please select 1 option)
LiDCO (lithium dilution cardiac output)
Oesophageal Doppler monitoring
PiCCO (pulse contour cardiac output)
Pulmonary artery flotation catheter (PAFC)
Transoesophageal echocardiography (TOE)

Score:
Submit answer

Skip question

Go to summary

Total Answered:

Question Navigator
1

10

11

12

13

14

15

16

17

18

19

20

21

22

23

24

25

26

27

28

29

30

31

32

33

34

35

36

37

38

39

40

41

42

43

Tags
Tag
Tag

Hide Question

2014 BMJ Publishing Group Ltd . All rights reserved.


Contact . Corporate . Terms and conditions & Privacy policy

My Profile

Exam Revision
Work Smart

Scores

Work Hard

Community
Mock Tests

Sign Out

Help
Group Learning 7

Revision Advice

Tags

Learning Journal

Work Smart
A 66-year-old male in intensive care has received 2000 mls of colloid in three hours.

Answer Statistics

He is receiving an escalating noradrenaline infusion (currently at 0.76 mcg/kg/min), has a blood pressure of
90/50 mmHg, pulse of 90 beats per min, a central venous pressure of 10 mmHg and capillary refill time of 2-3
seconds. His plasma lactate concentration is 2.9 mmol/L (<1.5).

Test Analysis

Which of the following is an appropriate method of measuring adequate intravascular filling?


(Please select 1 option)
LiDCO (lithium dilution cardiac output)

Incorrect answer selected

Oesophageal Doppler monitoring


PiCCO (pulse contour cardiac output)

This is the correct answer

Pulmonary artery flotation catheter (PAFC)


Transoesophageal echocardiography (TOE)

PiCCO gives indications of cardiac output, extravascular lung water, intravascular filling and only requires a
central line and a PiCCO femoral arterial line and as such is relatively simple to use.

Score:

It would also not be unreasonable to insert an oesophageal Doppler device; however they have greater
interobserver variation and require a degree of experience to use and are prone to misplacement.

Total Answered:

LiDCO is still not validated or practical to be widely used.

Feedback

PAFC are used widely in the USA however a study has questioned their safety and they have fallen out of
favour in the UK.

Rate this question

TOE requires considerable expertise to use and is not suitable for cardiac output studies.

Leave question feedback

Question Navigator
Next question

Related Articles (BMJ)

Go to summary

10

11

12

13

14

15

16

17

18

19

20

21

22

23

24

25

26

27

28

29

30

31

32

33

34

35

36

37

38

39

40

41

42

43

Revision Notes
Any revision notes recorded
here will be automatically
added to a list, and can be

My Profile

Exam Revision
Work Smart

Scores

Work Hard

Community
Mock Tests

Sign Out

Help
Group Learning 7

Revision Advice

Tags

Learning Journal

Work Smart
An 18-year-old male is admitted with a history of diarrhoea and vomiting associated with weakness and

Test Analysis

lethargy.
His motor power in the distal arms and legs is decreased and he describes difficulty swallowing. His forced vital
capacity (FVC) is 1.5 litres.
Which of the following is the most appropriate immediate treatment for this condition?
(Please select 1 option)
Cyclophosphamide
Intravenous immunoglobulin therapy (IgG) 0.5 g/kg
Intubation and ventilation
Plasmapheresis
Prednisolone 60 mg

Score:
Submit answer

Skip question

Go to summary

Total Answered:

Question Navigator
1

10

11

12

13

14

15

16

17

18

19

20

21

22

23

24

25

26

27

28

29

30

31

32

33

34

35

36

37

38

39

40

41

42

43

Tags
Tag
Tag

Hide Question

2014 BMJ Publishing Group Ltd . All rights reserved.


Contact . Corporate . Terms and conditions & Privacy policy

My Profile

Exam Revision
Work Smart

Scores

Work Hard

Community
Mock Tests

Sign Out

Help
Group Learning 7

Revision Advice

Tags

Learning Journal

Work Smart
An 18-year-old male is admitted with a history of diarrhoea and vomiting associated with weakness and

Answer Statistics

lethargy.
His motor power in the distal arms and legs is decreased and he describes difficulty swallowing. His forced vital
capacity (FVC) is 1.5 litres.

Test Analysis

Which of the following is the most appropriate immediate treatment for this condition?
(Please select 1 option)
Cyclophosphamide

Incorrect answer selected

Intravenous immunoglobulin therapy (IgG) 0.5 g/kg

This is the correct answer

Intubation and ventilation


Plasmapheresis
Prednisolone 60 mg

This scenario is suggestive of Guillain-Barre syndrome (GBS).


Dysphagia is a dangerous symptom suggestive of bulbar involvement. However a FVC of 1.5 litres is not an
indication for immediate ventilation (less than 1 litre is).

Score:
Total Answered:

First line therapy is intravenous IgG.

Feedback
Plasmapheresis can also be used but requires specialist equipment.

Rate this question

Steroids are of no benefit.


Cyclophosphamide although an immune suppressant is a red herring.

Leave question feedback

Question Navigator
Next question

Related Articles (BMJ)

Go to summary

10

11

12

13

14

15

16

17

18

19

20

21

22

23

24

25

26

27

28

29

30

31

32

33

34

35

36

37

38

39

40

41

42

43

Revision Notes
Any revision notes recorded
here will be automatically
added to a list, and can be

My Profile

Exam Revision
Work Smart

Scores

Work Hard

Community
Mock Tests

Sign Out

Help
Group Learning 7

Revision Advice

Tags

Learning Journal

Work Smart
A 21-year-old male is admitted with acute onset headache and is drowsy.

Test Analysis

He is opening his eyes spontaneously, is disoriented but is localising to painful stimuli.


Which of the following is the investigation of choice for this man?
(Please select 1 option)
Computed tomography (CT)
Lumbar puncture (LP)
Magnetic resonance angiography (MRA)
Magnetic resonance imaging (MRI)
Positron emission tomography (PET)

Submit answer

Skip question

Go to summary

Score:
Total Answered:

Question Navigator
1

10

11

12

13

14

15

16

17

18

19

20

21

22

23

24

25

26

27

28

29

30

31

32

33

34

35

36

37

38

39

40

41

42

43

Tags
Tag
Tag

Hide Question

2014 BMJ Publishing Group Ltd . All rights reserved.


Contact . Corporate . Terms and conditions & Privacy policy

My Profile

Exam Revision
Work Smart

Scores

Work Hard

Community
Mock Tests

Sign Out

Help
Group Learning 7

Revision Advice

Tags

Learning Journal

Work Smart
A 21-year-old male is admitted with acute onset headache and is drowsy.

Answer Statistics

He is opening his eyes spontaneously, is disoriented but is localising to painful stimuli.


Which of the following is the investigation of choice for this man?

Test Analysis

(Please select 1 option)


Computed tomography (CT)

Correct

Lumbar puncture (LP)


Magnetic resonance angiography (MRA)
Magnetic resonance imaging (MRI)
Positron emission tomography (PET)

Urgent CT will confirm diagnosis in 95% of patients with subarachnoid haemorrhage.


Lumbar puncture is not usually required unless the history is suggestive and the CT is normal.
Blood will be present in the cerebro-spinal fluid (if a bloody tap is suspected the number of red blood cells
should fall with each successive sample).
If the LP is performed six hours after onset of symptoms the supernatant fluid should be examined for
xanthochromia after centrifugation.

Score:
Total Answered:

Feedback
Rate this question

Next question

Go to summary

Leave question feedback

Related Articles (BMJ)

Question Navigator
1

10

11

12

13

14

15

16

17

18

19

20

21

22

23

24

25

26

27

28

29

30

31

32

33

34

35

36

37

38

39

40

41

42

43

Revision Notes
Any revision notes recorded
here will be automatically
added to a list, and can be

My Profile

Exam Revision
Work Smart

Scores

Work Hard

Community
Mock Tests

Sign Out

Help
Group Learning 7

Revision Advice

Tags

Learning Journal

Work Smart
A 21-year-old male is admitted with acute onset headache and is drowsy.

Test Analysis

He is opening his eyes spontaneously, is disoriented and is localising painful stimuli. He has a normal
computed tomography (CT) scan.
Which of the following is themost appropriate next investigation for this patient?
(Please select 1 option)
Cerebral angiography
Lumbar puncture
Magnetic resonance angiography
Magnetic resonance imaging
No further investigations necessary

Score:
Submit answer

Skip question

Go to summary

Total Answered:

Question Navigator
1

10

11

12

13

14

15

16

17

18

19

20

21

22

23

24

25

26

27

28

29

30

31

32

33

34

35

36

37

38

39

40

41

42

43

Tags
Tag
Tag

Hide Question

2014 BMJ Publishing Group Ltd . All rights reserved.


Contact . Corporate . Terms and conditions & Privacy policy

My Profile

Exam Revision
Work Smart

Scores

Work Hard

Community
Mock Tests

Sign Out

Help
Group Learning 7

Revision Advice

Tags

Learning Journal

Work Smart
A 21-year-old male is admitted with acute onset headache and is drowsy.

Answer Statistics

He is opening his eyes spontaneously, is disoriented and is localising painful stimuli. He has a normal
computed tomography (CT) scan.

Test Analysis

Which of the following is themost appropriate next investigation for this patient?
(Please select 1 option)
Cerebral angiography
Lumbar puncture

Incorrect answer selected


This is the correct answer

Magnetic resonance angiography


Magnetic resonance imaging
No further investigations necessary

Lumbar puncture (LP) is not usually required unless the history is suggestive and theCT is normal as in this
case.
Blood will be present in the CSF (if a bloody tap is suspected the number of red blood cells should fall with
each successive sample).
If the LP is performed six hours after onset of symptoms the supernatant fluid should be examined for
xanthochromia after centrifugation.

Score:
Total Answered:

Feedback
Rate this question

Next question

Go to summary

Leave question feedback

Related Articles (BMJ)

Question Navigator
1

10

11

12

13

14

15

16

17

18

19

20

21

22

23

24

25

26

27

28

29

30

31

32

33

34

35

36

37

38

39

40

41

42

43

Revision Notes
Any revision notes recorded
here will be automatically
added to a list, and can be

My Profile

Exam Revision
Work Smart

Scores

Work Hard

Community
Mock Tests

Sign Out

Help
Group Learning 7

Revision Advice

Tags

Learning Journal

Work Smart
A 67-year-old male is admitted with central chest pain of sudden onset which radiates through to his back.

Test Analysis

His blood pressure is 160/70 mmHg in his right arm and 140/60 mmHg in his left arm. He has
electrocardiographic (ECG) changes in leads II, III and AVF showing ST elevation of 2 mm.
What is the most likely diagnosis?
(Please select 1 option)
Coarctation of the aorta
Dissecting thoracic aortic aneurysm
Inferior myocardial infarct
Pancreatitis
Perforated duodenal ulcer

Score:
Submit answer

Skip question

Go to summary

Total Answered:

Question Navigator
1

10

11

12

13

14

15

16

17

18

19

20

21

22

23

24

25

26

27

28

29

30

31

32

33

34

35

36

37

38

39

40

41

42

43

Tags
Tag
Tag

Hide Question

2014 BMJ Publishing Group Ltd . All rights reserved.


Contact . Corporate . Terms and conditions & Privacy policy

My Profile

Exam Revision
Work Smart

Scores

Work Hard

Community
Mock Tests

Sign Out

Help
Group Learning 7

Revision Advice

Tags

Learning Journal

Work Smart
A 67-year-old male is admitted with central chest pain of sudden onset which radiates through to his back.
His blood pressure is 160/70 mmHg in his right arm and 140/60 mmHg in his left arm. He has
electrocardiographic (ECG) changes in leads II, III and AVF showing ST elevation of 2 mm.

Answer Statistics
Test Analysis

What is the most likely diagnosis?


(Please select 1 option)
Coarctation of the aorta

Incorrect answer selected

Dissecting thoracic aortic aneurysm

This is the correct answer

Inferior myocardial infarct


Pancreatitis
Perforated duodenal ulcer

This history is suggestive of a dissecting thoracic aortic aneurysm.


The ECG changes of inferior myocardial infarct suggest that the aneurysm has dissected the right coronary
artery at its ascending aortic ostium. An ascending aortic dissection needs immediate surgery. Whilst en route
to surgery, beta-blockade to control hypertension is appropriate.
An inferior myocardial infarct is high in the differential - however thrombolysis will kill a patient with an aortic
dissection.
Coarctation can give different blood pressures in either arm but is a chronic condition. Ulcer and pancreatitis
may mimic a heart attack, but with a normal ECG.

Score:
Total Answered:

Feedback
Rate this question
Leave question feedback

Next question

Go to summary

Question Navigator

Related Articles (BMJ)

10

11

12

13

14

15

16

17

18

19

20

21

22

23

24

25

26

27

28

29

30

31

32

33

34

35

36

37

38

39

40

41

42

43

Revision Notes
Any revision notes recorded
here will be automatically
added to a list, and can be

My Profile

Exam Revision
Work Smart

Scores

Work Hard

Community
Mock Tests

Sign Out

Help
Group Learning 7

Revision Advice

Tags

Learning Journal

Work Smart
A 27-year-old female with adult respiratory distress syndrome (ARDS) is ventilated on intensive care.

Test Analysis

Her inspired oxygen is 100%, positive end expiratory pressure is 15 cmH2O and peak airway pressure is 40
cmH2O.
Her arterial blood gas shows:
PaO 2

6 kPa

(11.3-12.6)

PaCO2

6.9 kPa

(4.7-6.0)

SpO2

88%

(>92%)

What treatment has been shown to decrease mortality in this patient group?
(Please select 1 option)
High frequency oscillatory ventilation (HFOV)
Increasing tidal volume and respiratory rate on the ventilator

Score:
Inhaled nitric oxide therapy

Total Answered:
Prone position

Question Navigator

Extracorporeal membrane oxygenation (ECMO)

Submit answer

Skip question

Go to summary

10

11

12

13

14

15

16

17

18

19

20

21

22

23

24

25

26

27

28

29

30

31

32

33

34

35

36

37

38

39

40

41

42

43

Tags
Tag
Tag

Hide Question

2014 BMJ Publishing Group Ltd . All rights reserved.


Contact . Corporate . Terms and conditions & Privacy policy

My Profile

Exam Revision
Work Smart

Scores

Work Hard

Community
Mock Tests

Sign Out

Help
Group Learning 7

Revision Advice

Tags

Learning Journal

Work Smart
A 27-year-old female with adult respiratory distress syndrome (ARDS) is ventilated on intensive care.

Answer Statistics

Her inspired oxygen is 100%, positive end expiratory pressure is 15 cmH2O and peak airway pressure is 40
cmH2O.

Test Analysis

Her arterial blood gas shows:


PaO 2

6 kPa

(11.3-12.6)

PaCO2

6.9 kPa

(4.7-6.0)

SpO2

88%

(>92%)

What treatment has been shown to decrease mortality in this patient group?
(Please select 1 option)
High frequency oscillatory ventilation (HFOV)

Incorrect answer selected

Increasing tidal volume and respiratory rate on the ventilator


Inhaled nitric oxide therapy
Prone position
Extracorporeal membrane oxygenation (ECMO)

Score:
This is the correct answer

This lady is on maximal ventilatory therapy but is still hypoxic. Her high CO2 is a reflection of permissive

Total Answered:

Feedback

hypercapnia to prevent overdistension of the lungs with high tidal volumes.

Rate this question


Treatment of these patients used to be extremely difficult, with no significant improvement in prognosis seen
will any technique. However, in 2010 the CESAR trial demonstrated a significant increase in survival without
significant disability with the use of extracorporeal membrane oxygenation (ECMO).
ECMO involves connecting a patient's circulation to an external oxygenator and pump, via a catheter placed in
the right side of the heart. It requires the continuous infusion of anticoagulant, and as such bleeding is the most
commonly associated complication. Infection and haemolysis are also a risk.
A recent study 1 published in the NEJM concluded that prone positioning improved outcome in severe ARDS.
However, it is only one study in a controversial area. There are a number of studies, including one by the same
group, which have not shown a mortality benefit (and others which show a detrimental effect on mortality).
Prone positioning is therefore not yet widely accepted in clinical practice, although this may change in the
future. For the purpose of the MRCP (where there are single best answers), ECMO remains the correct answer.
Reference:
1. Gurin C, Reignier J, Richard JC, et al. Prone positioning in severe acute respiratory distress syndrome. N
Engl J Med. 2013;368(23):2159-68.
2. Peek GJ, et al. Randomised controlled trial and parallel economic evaluation of conventional ventilatory
support versus extracorporeal membrane oxygenation for severe adult respiratory failure (CESAR). Health
Technol Assess. 2010;14(35):1-46.

Leave question feedback

Question Navigator
1

10

11

12

13

14

15

16

17

18

19

20

21

22

23

24

25

26

27

28

29

30

31

32

33

34

35

36

37

38

39

40

41

42

43

Revision Notes
Any revision notes recorded
here will be automatically
added to a list, and can be

My Profile

Exam Revision
Work Smart

Scores

Work Hard

Community
Mock Tests

Sign Out

Help
Group Learning 7

Work Smart

Revision Advice

Tags

Learning Journal

MRCP Part 1 2006 May

A 47-year-old man presents with progressive right hand swelling two days after being bitten by a dog.

Test Analysis

On examination there is a puncture wound with pus over the dorsum of the hand, cellulitis, ascending
lymphangitis and tender axillary lymphadenopathy.
What is the most appropriate antibiotics therapy in this case?
(Please select 1 option)
Benzylpenicillin and flucloxacillin
Ceftriaxone
Ciprofloxacin
Co-amoxiclav
Erythromycin

Score:
Submit answer

Skip question

Go to summary

Total Answered:

Question Navigator
1

10

11

12

13

14

15

16

17

18

19

20

21

22

23

24

25

26

27

28

29

30

31

32

33

34

35

36

37

38

39

40

41

42

43

Tags
Tag
Tag

Hide Question

2014 BMJ Publishing Group Ltd . All rights reserved.


Contact . Corporate . Terms and conditions & Privacy policy

My Profile

Exam Revision
Work Smart

Scores

Work Hard

Community
Mock Tests

Sign Out

Help
Group Learning 3

Work Smart

Revision Advice

Tags

Learning Journal

MRCP Part 1 2006 May

A 47-year-old man presents with progressive right hand swelling two days after being bitten by a dog.
On examination there is a puncture wound with pus over the dorsum of the hand, cellulitis, ascending
lymphangitis and tender axillary lymphadenopathy.

Answer Statistics
Test Analysis

What is the most appropriate antibiotics therapy in this case?


(Please select 1 option)
Benzylpenicillin and flucloxacillin

Incorrect answer selected

Ceftriaxone
Ciprofloxacin
Co-amoxiclav

This is the correct answer

Erythromycin

Only 15-20% of dog bites become infected, and providing the wound is appropriately cleaned and not
considered at risk (for example,crush or deep wounds) then antibiotic prophylaxis may not be required.
However, this patient has an infected wound, and infective organisms include Pasteurella spp, Staph. aureus
and anaerobes like Corynebacterium.

Score:
Total Answered:

The most appropriate antibiotic therapy in dog bites associated with cellulitis would be co-amoxiclav.

Feedback
Next question

Go to summary

Rate this question


Leave question feedback

Related Articles (BMJ)

Question Navigator
1

10

11

12

13

14

15

16

17

18

19

20

21

22

23

24

25

26

27

28

29

30

31

32

33

34

35

36

37

38

39

40

41

42

43

Revision Notes
Any revision notes recorded
here will be automatically
added to a list, and can be

My Profile

Exam Revision
Work Smart

Scores

Work Hard

Community
Mock Tests

Sign Out

Help
Group Learning 3

Revision Advice

Tags

Learning Journal

Work Smart
A 56-year-old man diagnosed with systemic inflammatory response syndrome (SIRS) secondary to pneumonia

Test Analysis

is admitted to the high dependency unit.


On examination, he has a temperature of 39C, a respiratory rate of 30/min, has a pulse of 109 beats/min and
a blood pressure of 89/74 mmHg despite receiving IV fluids and urine output of 25 ml/hour after catheterisation.
Which of the following should be instituted immediately and should be accomplished within the first six hours of
presentation?
(Please select 1 option)
Administer drotrecogin alfa (activated protein C)
Administer intravenous furosemide
Administer low dose steroids
Institute tight glucose control
Obtain blood cultures prior to antibiotic administration

Score:
Total Answered:

Submit answer

Skip question

Go to summary

Question Navigator
1

10

11

12

13

14

15

16

17

18

19

20

21

22

23

24

25

26

27

28

29

30

31

32

33

34

35

36

37

38

39

40

41

42

43

Tags
Tag
Tag

Hide Question

2014 BMJ Publishing Group Ltd . All rights reserved.


Contact . Corporate . Terms and conditions & Privacy policy

My Profile

Exam Revision
Work Smart

Scores

Work Hard

Community
Mock Tests

Sign Out

Help
Group Learning 3

Revision Advice

Tags

Learning Journal

Work Smart
A 56-year-old man diagnosed with systemic inflammatory response syndrome (SIRS) secondary to pneumonia

Answer Statistics

is admitted to the high dependency unit.


On examination, he has a temperature of 39C, a respiratory rate of 30/min, has a pulse of 109 beats/min and
a blood pressure of 89/74 mmHg despite receiving IV fluids and urine output of 25 ml/hour after catheterisation.

Test Analysis

Which of the following should be instituted immediately and should be accomplished within the first six hours of
presentation?
(Please select 1 option)
Administer drotrecogin alfa (activated protein C)

Incorrect answer selected

Administer intravenous furosemide


Administer low dose steroids
Institute tight glucose control
Obtain blood cultures prior to antibiotic administration

This is the correct answer

The Surviving Sepsis Campaign (a partnership of the Society of Critical Care Medicine, the European Society
of Intensive Care Medicine, and the International Sepsis Forum) has teamed up with the Institute for
Healthcare Improvement to develop severe sepsis bundles. A 'bundle' is a group of interventions related to a
disease process that, when executed together, result in better outcomes than when implemented individually.
Sepsis Resuscitation Bundle
(Should begin immediately, but must be accomplished within the first six hours of presentation.)
1. Serum lactate measured.
2. Blood cultures obtained prior to antibiotic administration.
3. From the time of presentation, broad-spectrum antibiotics administered within three hours for ED
admissions and one hour for non-ED ICU admissions.
4. In the event of hypotension and/or lactate > 4 mmol/l (36 mg/dl):
Deliver an initial minimum of 20 ml/kg of crystalloid (or colloid equivalent).
Apply vasopressors for hypotension not responding to initial fluid resuscitation to maintain mean arterial
pressure (MAP) > 65 mm Hg.
5. In the event of persistent hypotension despite fluid resuscitation (septic shock) and/or lactate > 4 mmol/l
(36 mg/dl):
Achieve central venous pressure (CVP) of > 8 mm Hg.
Achieve central venous oxygen saturation (ScvO2) of > 70%.
Sepsis Management Bundle
(To be accomplished as soon as possible may be completed within twenty-four hours of presentation.)

Score:
Total Answered:

Feedback
Rate this question
Leave question feedback

Question Navigator
1

10

11

12

13

14

15

16

17

18

19

20

21

22

23

24

25

26

27

28

29

30

31

32

33

34

35

36

37

38

39

40

41

42

43
1. Low-dose steroids administered for septic shock in accordance with a standardised ICU policy.
2. Glucose control maintained > lower limit of normal, but < 150 mg/dl (8.3 mmol/l).
3. Inspiratory plateau pressures maintained < 30 cm H 2 O for mechanically ventilated patients
Drotrectogin alpha (activated protein C) used to be recommended by NICE for the treatment of severe sepsis.
However, in October 2011 the company withdrew this from the market following the results of the PROWESSSHOCK study, which showed there was no statistically significant reduction in 28-day all-cause mortality in

Revision Notes
Any revision notes recorded
here will be automatically
added to a list, and can be

patients with septic shock.

viewed in full on the summary


page.

Reference:
Dellinger RP, et al. Surviving Sepsis Campaign: International guidelines for management of severe sepsis and
septic shock. Intensive Care Med. 2008;34(1):17-60.

Next question

Go to summary

Related Articles (BMJ)


Save

Tags
Tag
Tag

Hide Question

2014 BMJ Publishing Group Ltd . All rights reserved.


Contact . Corporate . Terms and conditions & Privacy policy

My Profile

Exam Revision
Work Smart

Scores

Work Hard

Community
Mock Tests

Sign Out

Help
Group Learning 3

Revision Advice

Tags

Learning Journal

Work Smart
A 78-year-old male who presents with increasing dysphagia is diagnosed with an inoperable carcinoma of the

Test Analysis

distal oesophagus. Oesophageal spasm causes food to stick after swallowing which causes odynophagia.
Which drug would be most helpful in relieving his chronic pain?
(Please select 1 option)
Clodronate
Dexamethasone
Nifedipine
Oxybutynin
Pinaverium

Submit answer

Skip question

Go to summary

Score:
Total Answered:

Question Navigator
1

10

11

12

13

14

15

16

17

18

19

20

21

22

23

24

25

26

27

28

29

30

31

32

33

34

35

36

37

38

39

40

41

42

43

Tags
Tag
Tag

Hide Question

2014 BMJ Publishing Group Ltd . All rights reserved.


Contact . Corporate . Terms and conditions & Privacy policy

My Profile

Exam Revision
Work Smart

Scores

Work Hard

Community
Mock Tests

Sign Out

Help
Group Learning 3

Revision Advice

Tags

Learning Journal

Work Smart
A 78-year-old male who presents with increasing dysphagia is diagnosed with an inoperable carcinoma of the

Answer Statistics

distal oesophagus. Oesophageal spasm causes food to stick after swallowing which causes odynophagia.
Which drug would be most helpful in relieving his chronic pain?

Test Analysis

(Please select 1 option)


Clodronate

Incorrect answer selected

Dexamethasone
Nifedipine

This is the correct answer

Oxybutynin
Pinaverium

Nifedipine helps relieve painful oesophageal spasm and tenesmus associated with gastrointestinal tumours and
could be used to relieve his odynophagia.
Clodronate inhibits osteoclastic bone resorption and is used to treat malignant bone pain and the associated
hypercalcaemia.

Score:

Pinaverium is used to reduce the pain duration associated with irritable bowel syndrome (IBS).

Total Answered:

Corticosteroids are used to treat pain from central nervous system tumours and painful bladder spasm may be
relieved by oxybutynin.

Feedback
Rate this question

Next question

Go to summary

Leave question feedback

Question Navigator
Related Articles (BMJ)

10

11

12

13

14

15

16

17

18

19

20

21

22

23

24

25

26

27

28

29

30

31

32

33

34

35

36

37

38

39

40

41

42

43

Revision Notes
Any revision notes recorded
here will be automatically
added to a list, and can be

My Profile

Exam Revision
Work Smart

Scores

Work Hard

Community
Mock Tests

Sign Out

Help
Group Learning 3

Revision Advice

Tags

Learning Journal

Work Smart
A 54-year-old obese man with a history of angina and hypertension presents with central crushing chest pain

Test Analysis

of two hours duration.


High flow oxygen, sublingual GTN and aspirin are administered and venous access is obtained. Whilst being
attached to an ECG monitor he collapses, with a doctor present, and the initial rhythm is pulseless ventricular
tachycardia (VT). The external defibrillator is located two minutes away on another ward.
Which of the following is the most appropriate immediate treatment for this man?
(Please select 1 option)
A ventilation to compression ratio of 30:2 should commenced
Await arrival of defibrillator, then deliver shock
Continuous chest compressions should be started
He should be given a praecordial thump
Intravenous adrenaline should be given

Score:
Total Answered:

Submit answer

Skip question

Go to summary

Question Navigator
1

10

11

12

13

14

15

16

17

18

19

20

21

22

23

24

25

26

27

28

29

30

31

32

33

34

35

36

37

38

39

40

41

42

43

Tags
Tag
Tag

Hide Question

2014 BMJ Publishing Group Ltd . All rights reserved.


Contact . Corporate . Terms and conditions & Privacy policy

My Profile

Exam Revision
Work Smart

Scores

Work Hard

Community

Help
Group Learning 3

Mock Tests

Sign Out

Revision Advice

Tags

Learning Journal

Work Smart
A 54-year-old obese man with a history of angina and hypertension presents with central crushing chest pain

Answer Statistics

of two hours duration.


High flow oxygen, sublingual GTN and aspirin are administered and venous access is obtained. Whilst being
attached to an ECG monitor he collapses, with a doctor present, and the initial rhythm is pulseless ventricular

Test Analysis

tachycardia (VT). The external defibrillator is located two minutes away on another ward.
Which of the following is the most appropriate immediate treatment for this man?
(Please select 1 option)
A ventilation to compression ratio of 30:2 should commenced

Incorrect answer selected

Await arrival of defibrillator, then deliver shock


Continuous chest compressions should be started
He should be given a praecordial thump

This is the correct answer

Intravenous adrenaline should be given

The 2010 ALS guidelines state that if a patient has a monitored and witnessed VF/VT arrest in hospital, three
quick successive (stacked) shocks should be given. Chest compressions should be started immediately after
the third, with a compression to ventilation ratio of 30:2 for 2 minutes.
A precordial thump can be successful if given within seconds of the onset of a shockable rhythm. Delivery
should not delay calling for help, or accessing a defibrillator, but would be indicated here whilst awaiting the
defibrillator. Chest compressions should start immediately if it is unsuccessful.

Score:
Total Answered:

Feedback
Rate this question

Intravenous adrenaline would be given every 3-5 minutes once chest compressions had started.
Leave question feedback
Next question

Related Articles (BMJ)

Go to summary

Question Navigator
1

10

11

12

13

14

15

16

17

18

19

20

21

22

23

24

25

26

27

28

29

30

31

32

33

34

35

36

37

38

39

40

41

42

43

Revision Notes
Any revision notes recorded
here will be automatically
added to a list, and can be

My Profile

Exam Revision
Work Smart

Scores

Work Hard

Community
Mock Tests

Sign Out

Help
Group Learning 3

Work Smart

Revision Advice

Tags

Learning Journal

MRCP Part 1 2006 Jan

A 23-year-old man with known peanut allergy presented to the Emergency department with anaphylaxis. He

Test Analysis

has a swollen face and lips.


His BP is 90/60 mmHg, pulse 110 bpm and he is wheezy.
Which of the following formulations of adrenaline should be given?
(Please select 1 option)
0.5 ml of 1:10000 adrenaline IM
0.5 ml of 1:1000 adrenaline IM
5 ml of 1:1000 adrenaline IM
10 ml of 1:10000 adrenaline IV
Nebulised adrenaline

Score:
Submit answer

Skip question

Go to summary

Total Answered:

Question Navigator
1

10

11

12

13

14

15

16

17

18

19

20

21

22

23

24

25

26

27

28

29

30

31

32

33

34

35

36

37

38

39

40

41

42

43

Tags
Tag
Tag

Hide Question

2014 BMJ Publishing Group Ltd . All rights reserved.


Contact . Corporate . Terms and conditions & Privacy policy

My Profile

Exam Revision
Work Smart

Scores

Work Hard

Community
Mock Tests

Sign Out

Help
Group Learning 3

Work Smart

Revision Advice

Tags

Learning Journal

MRCP Part 1 2006 Jan

A 23-year-old man with known peanut allergy presented to the Emergency department with anaphylaxis. He

Answer Statistics

has a swollen face and lips.


His BP is 90/60 mmHg, pulse 110 bpm and he is wheezy.

Test Analysis

Which of the following formulations of adrenaline should be given?


(Please select 1 option)
0.5 ml of 1:10000 adrenaline IM
0.5 ml of 1:1000 adrenaline IM

Incorrect answer selected


This is the correct answer

5 ml of 1:1000 adrenaline IM
10 ml of 1:10000 adrenaline IV
Nebulised adrenaline

For adults, a dose of 0.5 mL adrenaline 1:1000 solution (500 micrograms) should be administered
intramuscularly, and repeated after about five minutes in the absence of clinical improvement or if deterioration
occurs after the initial treatment especially if consciousness becomes - or remains - impaired as a result of
hypotension.

Score:
Total Answered:

The intramuscular (IM) route for adrenaline is the route of choice for most healthcare providers. There is a
much greater risk of causing harmful side effects by inappropriate dosage or misdiagnosis of anaphylaxis when
using IV adrenaline.
Adult EpiPen which allergy sufferers can carry with them contains 0.3 mg or 0.15 mg adrenaline in a 1:1000
dilution for intramuscular (IM) injection.

Feedback
Rate this question
Leave question feedback

Next question

Related Articles (BMJ)

Go to summary

Question Navigator
1

10

11

12

13

14

15

16

17

18

19

20

21

22

23

24

25

26

27

28

29

30

31

32

33

34

35

36

37

38

39

40

41

42

43

Revision Notes
Any revision notes recorded
here will be automatically
added to a list, and can be

My Profile

Exam Revision
Work Smart

Scores

Work Hard

Community
Mock Tests

Sign Out

Help
Group Learning 3

Revision Advice

Tags

Learning Journal

Work Smart
A 67-year-old man complains of dizziness and faintness.

Test Analysis

He has insulin dependent diabetes mellitus and he had a sigmoid colectomy 3 days previously . His blood
pressure is 80/50 mmHg, his pulse 110 beats per min, his respiratory rate 24/min, and he has SpO 2 99% on
air. His plasma glucose concentration is 18 mmol/L (3.0-6.0 Fasting)
Which of the following is the most appropriate investigation for this patient?
(Please select 1 option)
Arterial blood gas
Chest x ray
Electrocardiogram
Serum Lactate
Urine ketones

Score:
Total Answered:
Submit answer

Skip question

Go to summary

Question Navigator
1

10

11

12

13

14

15

16

17

18

19

20

21

22

23

24

25

26

27

28

29

30

31

32

33

34

35

36

37

38

39

40

41

42

43

Tags
Tag
Tag

Hide Question

2014 BMJ Publishing Group Ltd . All rights reserved.


Contact . Corporate . Terms and conditions & Privacy policy

My Profile

Exam Revision
Work Smart

Scores

Work Hard

Community
Mock Tests

Sign Out

Help
Group Learning 3

Revision Advice

Tags

Learning Journal

Work Smart
A 67-year-old man complains of dizziness and faintness.

Answer Statistics

He has insulin dependent diabetes mellitus and he had a sigmoid colectomy 3 days previously . His blood
pressure is 80/50 mmHg, his pulse 110 beats per min, his respiratory rate 24/min, and he has SpO 2 99% on
air. His plasma glucose concentration is 18 mmol/L (3.0-6.0 Fasting)

Test Analysis

Which of the following is the most appropriate investigation for this patient?
(Please select 1 option)
Arterial blood gas

Incorrect answer selected

Chest x ray
Electrocardiogram

This is the correct answer

Serum Lactate
Urine ketones

This man may have a cardiac cause for his dizziness. The highest prevalence of myocardial infarction (MI) is
72 hours post operation.
Patients with diabetes may not have chest pain due to autonomic dysfunction. The differential diagnosis would
include pulmonary embolus. It may also include diabetic ketoacidosis, but this would be unlikely with his
glucose at 18 mmol/L and would not directly explain his hypotension. Also, he would be expected to have a
slightly higher respiratory rate than 24/min.

Score:
Total Answered:

Feedback
Rate this question

The most appropriate immediate investigation in this scenario would be ECG.


Leave question feedback
Next question

Go to summary

Question Navigator

Related Articles (BMJ)

10

11

12

13

14

15

16

17

18

19

20

21

22

23

24

25

26

27

28

29

30

31

32

33

34

35

36

37

38

39

40

41

42

43

Revision Notes
Any revision notes recorded
here will be automatically
added to a list, and can be

My Profile

Exam Revision
Work Smart

Scores

Work Hard

Community
Mock Tests

Sign Out

Help
Group Learning 3

Revision Advice

Tags

Learning Journal

Work Smart
A patient needs central venous access for total parenteral nutrition (TPN).

Test Analysis

Which of the following is the cleanest site for placement?


(Please select 1 option)
Left femoral
Left internal jugular
Right femoral
Right internal jugular
Right subclavian

Submit answer

Skip question

Go to summary

Score:
Total Answered:

Question Navigator
1

10

11

12

13

14

15

16

17

18

19

20

21

22

23

24

25

26

27

28

29

30

31

32

33

34

35

36

37

38

39

40

41

42

43

Tags
Tag
Tag

Hide Question

2014 BMJ Publishing Group Ltd . All rights reserved.


Contact . Corporate . Terms and conditions & Privacy policy

My Profile

Exam Revision
Work Smart

Scores

Work Hard

Community
Mock Tests

Sign Out

Help
Group Learning 3

Revision Advice

Tags

Learning Journal

Work Smart
A patient needs central venous access for total parenteral nutrition (TPN).

Answer Statistics

Which of the following is the cleanest site for placement?

Test Analysis

(Please select 1 option)


Left femoral

Incorrect answer selected

Left internal jugular


Right femoral
Right internal jugular
Right subclavian

This is the correct answer

Right or left subclavian is regarded as the cleanest site for central venous access. It also the most tolerated by
patients.
However the incidence of subclinical pneumothorax even in the hands of experienced clinicians has led to it
falling out of favour.
Score:
Next question

Go to summary

Total Answered:

Feedback
Related Articles (BMJ)

Rate this question


Leave question feedback

Question Navigator
1

10

11

12

13

14

15

16

17

18

19

20

21

22

23

24

25

26

27

28

29

30

31

32

33

34

35

36

37

38

39

40

41

42

43

Revision Notes
Any revision notes recorded
here will be automatically
added to a list, and can be

My Profile

Exam Revision
Work Smart

Scores

Work Hard

Community
Mock Tests

Sign Out

Help
Group Learning 3

Revision Advice

Tags

Learning Journal

Work Smart
An 18-year-old with cerebral palsy is admitted after a respiratory arrest having been intubated by paramedics.

Test Analysis

Nobody can gain intravenous access as the patient is too shut down. A femoral line is not possible due to
contractures. You do not have the experience to perform central venous cannulation.
Which of the following is the best option for administering intravenous fluids/emergency drugs in this situation
of inability to gain venous access?
(Please select 1 option)
Down the endotracheal tube
Intramuscular (IM)
Intraosseous
Nasogastric
Subcutaneous

Score:
Submit answer

Skip question

Go to summary

Total Answered:

Question Navigator
1

10

11

12

13

14

15

16

17

18

19

20

21

22

23

24

25

26

27

28

29

30

31

32

33

34

35

36

37

38

39

40

41

42

43

Tags
Tag
Tag

Hide Question

2014 BMJ Publishing Group Ltd . All rights reserved.


Contact . Corporate . Terms and conditions & Privacy policy

My Profile

Exam Revision
Work Smart

Scores

Work Hard

Community
Mock Tests

Sign Out

Help
Group Learning 3

Revision Advice

Tags

Learning Journal

Work Smart
An 18-year-old with cerebral palsy is admitted after a respiratory arrest having been intubated by paramedics.
Nobody can gain intravenous access as the patient is too shut down. A femoral line is not possible due to
contractures. You do not have the experience to perform central venous cannulation.

Answer Statistics
Test Analysis

Which of the following is the best option for administering intravenous fluids/emergency drugs in this situation
of inability to gain venous access?
(Please select 1 option)
Down the endotracheal tube

Incorrect answer selected

Intramuscular (IM)
Intraosseous

This is the correct answer

Nasogastric
Subcutaneous

Nasogastric, IM and subcutaneous are too slow and unreliable for emergency situations (although in cardiac
arrest the endotracheal route is recognised).

Score:

Venous cut down is a possibility but requires skill in the procedure.

Total Answered:

Intraosseous is still perfectly viable in the adult patient 2 cm below the tibial tuberosity on the antero-medial
side or 2 cm proximal to the medial malleolus.

Feedback
Rate this question

Next question

Go to summary

Leave question feedback

Related Articles (BMJ)

Question Navigator
1

10

11

12

13

14

15

16

17

18

19

20

21

22

23

24

25

26

27

28

29

30

31

32

33

34

35

36

37

38

39

40

41

42

43

Revision Notes
Any revision notes recorded
here will be automatically
added to a list, and can be

My Profile

Exam Revision
Work Smart

Scores

Work Hard

Community
Mock Tests

Sign Out

Help
Group Learning 3

Revision Advice

Tags

Learning Journal

Work Smart
A 26-year-old female is admitted to ICU with severe asthma.

Test Analysis

She is ventilated for one week and receives IV co-amoxiclav/clarithromycin, magnesium, prednisolone,
sedatives and muscle relaxants.
She improves gradually but two days after stopping muscle relaxants she still is unable to be weaned from
ventilatory support.
On examination, she is alert but has flaccid weakness of all limbs.
Which of the following is the likely diagnosis?
(Please select 1 option)
Critical illness polyneuropathy
Guillain-Barr syndrome
Hypermagnesaemia
Prolonged neuromuscular blockade

Score:

Steroid induced myopathy

Total Answered:

Question Navigator
Submit answer

Skip question

Go to summary

10

11

12

13

14

15

16

17

18

19

20

21

22

23

24

25

26

27

28

29

30

31

32

33

34

35

36

37

38

39

40

41

42

43

Tags
Tag
Tag

Hide Question

2014 BMJ Publishing Group Ltd . All rights reserved.


Contact . Corporate . Terms and conditions & Privacy policy

My Profile

Exam Revision
Work Smart

Scores

Work Hard

Community
Mock Tests

Sign Out

Help
Group Learning 3

Revision Advice

Tags

Learning Journal

Work Smart
A 26-year-old female is admitted to ICU with severe asthma.

Answer Statistics

She is ventilated for one week and receives IV co-amoxiclav/clarithromycin, magnesium, prednisolone,
sedatives and muscle relaxants.

Test Analysis

She improves gradually but two days after stopping muscle relaxants she still is unable to be weaned from
ventilatory support.
On examination, she is alert but has flaccid weakness of all limbs.
Which of the following is the likely diagnosis?
(Please select 1 option)
Critical illness polyneuropathy

Incorrect answer selected

Guillain-Barr syndrome
Hypermagnesaemia
Prolonged neuromuscular blockade

This is the correct answer

Steroid induced myopathy

Score:
The history suggests prolonged neuromuscular junction (NMJ) blockade which may be exacerbated by both
corticosteroids and magnesium.

Total Answered:

Feedback

This condition was originally described with suxamethonium due to hereditary reductions in plasma
cholinesterase activity.

Rate this question

However, drugs and electrolyte abnormalities may exacerbate this.


Leave question feedback
Next question

Related Articles (BMJ)

Go to summary

Question Navigator
1

10

11

12

13

14

15

16

17

18

19

20

21

22

23

24

25

26

27

28

29

30

31

32

33

34

35

36

37

38

39

40

41

42

43

Revision Notes
Any revision notes recorded
here will be automatically
added to a list, and can be

My Profile

Exam Revision
Work Smart

Scores

Work Hard

Community
Mock Tests

Sign Out

Help
Group Learning 3

Revision Advice

Tags

Learning Journal

Work Smart
A patient post retrosternal thyroidectomy resection has sudden onset shortness of breath.

Test Analysis

On examination, she is talking clearly but has decreased breath sounds on her right side with hyper-resonance
on percussion. Her blood pressure is 110/80 mmHg, pulse 95 beats per minute, respiratory rate 24/min and
SpO 2 92 on air.
Which of the following would be most appropriate for her initially?
(Please select 1 option)
Chest x ray
Furosemide 40 mg intravenously
Intercostal chest drain insertion
Needle thoracocentesis
Removal of surgical clips

Score:
Submit answer

Skip question

Go to summary

Total Answered:

Question Navigator
1

10

11

12

13

14

15

16

17

18

19

20

21

22

23

24

25

26

27

28

29

30

31

32

33

34

35

36

37

38

39

40

41

42

43

Tags
Tag
Tag

Hide Question

2014 BMJ Publishing Group Ltd . All rights reserved.


Contact . Corporate . Terms and conditions & Privacy policy

My Profile

Exam Revision
Work Smart

Scores

Work Hard

Community
Mock Tests

Sign Out

Help
Group Learning 3

Revision Advice

Tags

Learning Journal

Work Smart
A patient post retrosternal thyroidectomy resection has sudden onset shortness of breath.

Answer Statistics

On examination, she is talking clearly but has decreased breath sounds on her right side with hyper-resonance
on percussion. Her blood pressure is 110/80 mmHg, pulse 95 beats per minute, respiratory rate 24/min and
SpO 2 92 on air.

Test Analysis

Which of the following would be most appropriate for her initially?


(Please select 1 option)
Chest x ray

Correct

Furosemide 40 mg intravenously
Intercostal chest drain insertion
Needle thoracocentesis
Removal of surgical clips

According to the scenario above this lady has a pneumothorax.


She is not in extremis (suggesting a tension pneumothorax) so needle thoracocentesis is not required.

Score:
Total Answered:

The history and examination are suggestive of a pneumothorax and therefore, with her being relatively stable,
the most appropriate first step would be confirmation with chest x ray followed by chest drain insertion.

Feedback
Furosemide would not help.

Rate this question


Next question

Go to summary

Leave question feedback

Question Navigator
Related Articles (BMJ)

10

11

12

13

14

15

16

17

18

19

20

21

22

23

24

25

26

27

28

29

30

31

32

33

34

35

36

37

38

39

40

41

42

43

Revision Notes
Any revision notes recorded
here will be automatically
added to a list, and can be

My Profile

Exam Revision
Work Smart

Scores

Work Hard

Community
Mock Tests

Sign Out

Help
Group Learning 3

Revision Advice

Tags

Learning Journal

Work Smart
A 10-year-old child presents with a respiratory tract infection. You decide to treat him empirically with a broad

Test Analysis

spectrum antibiotic.
The child is taking no other medication, has no other co-morbidity factors and has no known detected allergies.
Which of the following antibiotics is contraindicated in this patient?
(Please select 1 option)
Amoxicillin
Clarithromycin
Erythromycin
Flucloxacillin
Minocycline

Score:
Submit answer

Skip question

Go to summary

Total Answered:

Question Navigator
1

10

11

12

13

14

15

16

17

18

19

20

21

22

23

24

25

26

27

28

29

30

31

32

33

34

35

36

37

38

39

40

41

42

43

Tags
Tag
Tag

Hide Question

2014 BMJ Publishing Group Ltd . All rights reserved.


Contact . Corporate . Terms and conditions & Privacy policy

My Profile

Exam Revision
Work Smart

Scores

Work Hard

Community
Mock Tests

Sign Out

Help
Group Learning 3

Revision Advice

Tags

Learning Journal

Work Smart
A 10-year-old child presents with a respiratory tract infection. You decide to treat him empirically with a broad

Answer Statistics

spectrum antibiotic.
The child is taking no other medication, has no other co-morbidity factors and has no known detected allergies.

Test Analysis

Which of the following antibiotics is contraindicated in this patient?


(Please select 1 option)
Amoxicillin

Incorrect answer selected

Clarithromycin
Erythromycin
Flucloxacillin
Minocycline

This is the correct answer

Tetracyclines can bind to calcium and deposit on growing bones and teeth. This may cause staining and
occasionally dental hypoplasia.
Therefore, tetracyclines should not be given to children under the age of 12 or to pregnant or breastfeeding
women.

Score:
Total Answered:

However, doxycycline can be used (unlicenced) for the treatment of anthrax in children.

Feedback
BNF 60

Rate this question


Next question

Go to summary

Leave question feedback

Question Navigator
Related Articles (BMJ)

10

11

12

13

14

15

16

17

18

19

20

21

22

23

24

25

26

27

28

29

30

31

32

33

34

35

36

37

38

39

40

41

42

43

Revision Notes
Any revision notes recorded
here will be automatically
added to a list, and can be

My Profile

Exam Revision
Work Smart

Scores

Work Hard

Community
Mock Tests

Sign Out

Help
Group Learning 3

Revision Advice

Tags

Learning Journal

Work Smart
Mr YB is admitted on your ward with endocarditis and is prescribed vancomycin IV.

Test Analysis

You monitor the patient for signs of toxicity as it has a narrow therapeutic index.
Which of the following is a result of vancomycin toxicity?
(Please select 1 option)
Bradycardia
Dry mouth
Erythema multiforme
Hepatoxicity
Ototoxicity

Submit answer

Skip question

Go to summary

Score:
Total Answered:

Question Navigator
1

10

11

12

13

14

15

16

17

18

19

20

21

22

23

24

25

26

27

28

29

30

31

32

33

34

35

36

37

38

39

40

41

42

43

Tags
Tag
Tag

Hide Question

2014 BMJ Publishing Group Ltd . All rights reserved.


Contact . Corporate . Terms and conditions & Privacy policy

My Profile

Exam Revision
Work Smart

Scores

Work Hard

Community
Mock Tests

Sign Out

Help
Group Learning 3

Revision Advice

Tags

Learning Journal

Work Smart
Mr YB is admitted on your ward with endocarditis and is prescribed vancomycin IV.

Answer Statistics

You monitor the patient for signs of toxicity as it has a narrow therapeutic index.
Which of the following is a result of vancomycin toxicity?

Test Analysis

(Please select 1 option)


Bradycardia

Incorrect answer selected

Dry mouth
Erythema multiforme
Hepatoxicity
Ototoxicity

This is the correct answer

Ototoxicity is associated with vancomycin, and is more likely in patients with high plasma concentrations, or
with renal impairment or pre-existing hearing loss.
It may progress after drug withdrawal, and may be irreversible.Hearing loss may be preceded by tinnitus,
which must be regarded as a sign to stop treatment.
The important level to measure here is the trough level as opposed to the peak level with gentamicin.
(Martindale)

Score:
Total Answered:

Feedback
Rate this question

Next question

Go to summary

Leave question feedback

Related Articles (BMJ)

Question Navigator
1

10

11

12

13

14

15

16

17

18

19

20

21

22

23

24

25

26

27

28

29

30

31

32

33

34

35

36

37

38

39

40

41

42

43

Revision Notes
Any revision notes recorded
here will be automatically
added to a list, and can be

My Profile

Exam Revision
Work Smart

Scores

Work Hard

Community
Mock Tests

Sign Out

Help
Group Learning 3

Revision Advice

Tags

Learning Journal

Work Smart
Ms YF is prescribed ciprofloxacin 500 mg twice daily for the treatment of cystitis.

Test Analysis

In which of the following conditions should ciprofloxacin be used with caution?


(Please select 1 option)
Asthma
Diabetes
Epilepsy
Glaucoma
Heart failure

Submit answer

Skip question

Go to summary

Score:
Total Answered:

Question Navigator
1

10

11

12

13

14

15

16

17

18

19

20

21

22

23

24

25

26

27

28

29

30

31

32

33

34

35

36

37

38

39

40

41

42

43

Tags
Tag
Tag

Hide Question

2014 BMJ Publishing Group Ltd . All rights reserved.


Contact . Corporate . Terms and conditions & Privacy policy

My Profile

Exam Revision
Work Smart

Scores

Work Hard

Community
Mock Tests

Sign Out

Help
Group Learning 3

Revision Advice

Tags

Learning Journal

Work Smart
Ms YF is prescribed ciprofloxacin 500 mg twice daily for the treatment of cystitis.

Answer Statistics

In which of the following conditions should ciprofloxacin be used with caution?

Test Analysis

(Please select 1 option)


Asthma

Incorrect answer selected

Diabetes
Epilepsy

This is the correct answer

Glaucoma
Heart failure

Quinolones should be used with caution in patients with a history of epilepsy or conditions that predispose to
seizures, in glucose-6-phosphate dehydrogenase (G6PD) deficiency, myasthenia gravis (risk of exacerbation),
and in children or adolescents.
The CSM has warned that quinolones may induce convulsions in patients with or without a history of
convulsions; taking non-steroidal anti-inflammatory drugs (NSAIDs) at the same time may also induce them.

Score:

It should also be noted that ciprofloxacin is contraindicated in pregnancy.

Total Answered:

BNF 60

Next question

Feedback
Go to summary

Rate this question


Leave question feedback

Related Articles (BMJ)

Question Navigator
1

10

11

12

13

14

15

16

17

18

19

20

21

22

23

24

25

26

27

28

29

30

31

32

33

34

35

36

37

38

39

40

41

42

43

Revision Notes
Any revision notes recorded
here will be automatically
added to a list, and can be

My Profile

Exam Revision
Work Smart

Scores

Work Hard

Community
Mock Tests

Sign Out

Help
Group Learning 3

Revision Advice

Tags

Learning Journal

Work Smart
Mr UP is taking bendroflumethiazide for hypertension.

Test Analysis

Which of the following is likely to increase whilst on bendroflumethiazide therapy?

(Please select 1 option)


Magnesium
Potassium
Sodium
Uric acid
White cell count

Submit answer

Skip question

Go to summary

Score:
Total Answered:

Question Navigator
1

10

11

12

13

14

15

16

17

18

19

20

21

22

23

24

25

26

27

28

29

30

31

32

33

34

35

36

37

38

39

40

41

42

43

Tags
Tag
Tag

Hide Question

2014 BMJ Publishing Group Ltd . All rights reserved.


Contact . Corporate . Terms and conditions & Privacy policy

My Profile

Exam Revision
Work Smart

Scores

Work Hard

Community
Mock Tests

Sign Out

Help
Group Learning 3

Revision Advice

Tags

Learning Journal

Work Smart
Mr UP is taking bendroflumethiazide for hypertension.

Answer Statistics

Which of the following is likely to increase whilst on bendroflumethiazide therapy?

Test Analysis

(Please select 1 option)


Magnesium

Incorrect answer selected

Potassium
Sodium
Uric acid

This is the correct answer

White cell count

Hydrochlorothiazide and other thiazide diuretics may cause metabolic disturbances especially at high doses.
They may provoke hyperglycaemia and glycosuria in diabetic and other susceptible patients. They may cause
hyperuricaemia and precipitate attacks of gout in some patients.
Score:

Thiazide diuretics may be associated with electrolyte imbalances including hypochloraemic alkalosis,
hyponatraemia, and hypokalaemia.

Total Answered:

Hypokalaemia intensifies the effect of digitalis on cardiac muscle and treatment with digitalis or its glycosides
may have to be temporarily suspended. Patients with cirrhosis of the liver are particularly at risk from
hypokalaemia.

Feedback
Rate this question

Hyponatraemia may occur in patients with severe heart failure who are very oedematous, particularly with large
doses used with restricted salt in the diet.

Leave question feedback

The urinary excretion of calcium is reduced.Hypomagnesaemia has also occurred.

Question Navigator
Adverse changes in plasma lipids have also been noted but their clinical significance is unclear.
(Martindale)

Next question

Related Articles (BMJ)

Go to summary

10

11

12

13

14

15

16

17

18

19

20

21

22

23

24

25

26

27

28

29

30

31

32

33

34

35

36

37

38

39

40

41

42

43

Revision Notes
Any revision notes recorded
here will be automatically
added to a list, and can be

My Profile

Exam Revision
Work Smart

Scores

Work Hard

Community
Mock Tests

Sign Out

Help
Group Learning 3

Revision Advice

Tags

Learning Journal

Work Smart
Mrs PL is taking digoxin for heart failure.

Test Analysis

Which of the following drugs, if added to her prescription, would predispose Mrs PL to digoxin toxicity?
(Please select 1 option)
Bumetanide
Ibuprofen
Paracetamol
Phenytoin
St. John's wort

Submit answer

Skip question

Go to summary

Score:
Total Answered:

Question Navigator
1

10

11

12

13

14

15

16

17

18

19

20

21

22

23

24

25

26

27

28

29

30

31

32

33

34

35

36

37

38

39

40

41

42

43

Tags
Tag
Tag

Hide Question

2014 BMJ Publishing Group Ltd . All rights reserved.


Contact . Corporate . Terms and conditions & Privacy policy

My Profile

Exam Revision
Work Smart

Scores

Work Hard

Community
Mock Tests

Sign Out

Help
Group Learning 3

Revision Advice

Tags

Learning Journal

Work Smart
Mrs PL is taking digoxin for heart failure.

Answer Statistics

Which of the following drugs, if added to her prescription, would predispose Mrs PL to digoxin toxicity?

Test Analysis

(Please select 1 option)


Bumetanide

Correct

Ibuprofen
Paracetamol
Phenytoin
St. John's wort

Bumetanide is a loop diuretic and may cause hypokalaemia as a side effect. The potassium loss caused by
bumetanide increases the toxicity of digoxin.
A comparative study of the medical records of 418 patients taking digitalis over the period 1950 to 1952, and of
679 patients over the period 1964 to 1966, found that the incidence of digitalis toxicity had more than doubled.1
Of the earlier group 8.6% developed toxicity compared with 17.2% of the latter group (81% taking diuretics,
mainly chlorothiazides, furosemide, ethacrynic acid, chlortalidone). It was concluded that the increased toxicity
was related to the increased usage of potassium-depleting diuretics.

Score:
Total Answered:

Reference:

Feedback

1. Jorgensen AW, Sorensen OH. Digitalis intoxication. A comparative study on the incidence of digitalis

Rate this question

intoxication during the periods 1950-52 and 1964-66. Acta Med Scand. 1970;188(3):179-83.
Leave question feedback
Next question

Related Articles (BMJ)

Go to summary

Question Navigator
1

10

11

12

13

14

15

16

17

18

19

20

21

22

23

24

25

26

27

28

29

30

31

32

33

34

35

36

37

38

39

40

41

42

43

Revision Notes
Any revision notes recorded
here will be automatically
added to a list, and can be

My Profile

Exam Revision
Work Smart

Scores

Work Hard

Community
Mock Tests

Sign Out

Help
Group Learning 3

Revision Advice

Tags

Learning Journal

Work Smart
Mrs HV is taking an antidepressant.

Test Analysis

Her husband recently passed away and she was diagnosed as being clinically depressed.
Since taking the antidepressants, she has been complaining of drowsiness, confusion and fatigue.
Depletion of which of the following electrolytes may be causing Mrs HV's symptoms?
(Please select 1 option)
Chloride
Magnesium
Phosphate
Potassium
Sodium

Score:
Submit answer

Skip question

Go to summary

Total Answered:

Question Navigator
1

10

11

12

13

14

15

16

17

18

19

20

21

22

23

24

25

26

27

28

29

30

31

32

33

34

35

36

37

38

39

40

41

42

43

Tags
Tag
Tag

Hide Question

2014 BMJ Publishing Group Ltd . All rights reserved.


Contact . Corporate . Terms and conditions & Privacy policy

My Profile

Exam Revision
Work Smart

Scores

Work Hard

Community
Mock Tests

Sign Out

Help
Group Learning 3

Revision Advice

Tags

Learning Journal

Work Smart
Mrs HV is taking an antidepressant.

Answer Statistics

Her husband recently passed away and she was diagnosed as being clinically depressed.
Since taking the antidepressants, she has been complaining of drowsiness, confusion and fatigue.

Test Analysis

Depletion of which of the following electrolytes may be causing Mrs HV's symptoms?
(Please select 1 option)
Chloride

Incorrect answer selected

Magnesium
Phosphate
Potassium
Sodium

This is the correct answer

The Committee on Safety of Medicines (CSM) have reported thathyponatraemia is associated with all types of
antidepressants; however it has been reported more frequently with selective serotonin reuptake inhibitors
(SSRIs) than with other antidepressants.
Hyponatraemia should be considered in all patients who develop drowsiness, confusion or convulsions whilst
taking an antidepressant.

Score:
Total Answered:

Feedback

BNF 60

Rate this question


Next question

Go to summary

Leave question feedback

Question Navigator
Related Articles (BMJ)

10

11

12

13

14

15

16

17

18

19

20

21

22

23

24

25

26

27

28

29

30

31

32

33

34

35

36

37

38

39

40

41

42

43

Revision Notes
Any revision notes recorded
here will be automatically
added to a list, and can be

My Profile

Exam Revision
Work Smart

Scores

Work Hard

Community
Mock Tests

Sign Out

Help
Group Learning 4

Revision Advice

Tags

Learning Journal

Work Smart
You would like to prescribe a selective serotonin reuptake inhibitor (SSRI) for a 14-year-old girl who has been

Test Analysis

diagnosed as being clinically depressed.


After much debate and intervention from various healthcare professionals, it was decided to prescribe her
fluoxetine.
For which one of the following parameters should the patient be closely monitored, especially at the beginning
of treatment?
(Please select 1 option)
Coldness of extremities
Hostility
Hyperglycaemia
Prothrombin time
Tachycardia

Score:
Total Answered:

Submit answer

Skip question

Go to summary

Question Navigator
1

10

11

12

13

14

15

16

17

18

19

20

21

22

23

24

25

26

27

28

29

30

31

32

33

34

35

36

37

38

39

40

41

42

43

Tags
Tag
Tag

Hide Question

2014 BMJ Publishing Group Ltd . All rights reserved.


Contact . Corporate . Terms and conditions & Privacy policy

My Profile

Exam Revision
Work Smart

Scores

Work Hard

Community
Mock Tests

Sign Out

Help
Group Learning 5

Revision Advice

Tags

Learning Journal

Work Smart
You would like to prescribe a selective serotonin reuptake inhibitor (SSRI) for a 14-year-old girl who has been

Answer Statistics

diagnosed as being clinically depressed.


After much debate and intervention from various healthcare professionals, it was decided to prescribe her
fluoxetine.

Test Analysis

For which one of the following parameters should the patient be closely monitored, especially at the beginning
of treatment?
(Please select 1 option)
Coldness of extremities
Hostility

Incorrect answer selected

This is the correct answer

Hyperglycaemia
Prothrombin time
Tachycardia

Children and adolescents need to be monitored carefully. The use of antidepressants has been linked with
suicidal thoughts and behaviour. Where necessary patients should be monitored for suicidal behaviour, self
harm or hostility, particularly at the beginning of treatment or when the dose is changed.

Score:
Total Answered:

The balance of risks and benefits for the treatment of depressive illness in individuals under the age of 18
years is considered unfavourable for citalopram, escitalopram, paroxetine and sertraline.

Feedback

Clinical trials have failed to show efficacy and have shown an increase in harmful outcomes.

Rate this question

Only fluoxetine has shown to be effective in treating depressive illness in children, but careful monitoring for the
above sign is required.

Leave question feedback

Reference:

Question Navigator

electronic Medicines Compendium (eMC). Fluoxetine 20 mg Capsules


Joint Formulary Committee. British National Formulary 60. London: Pharmaceutical Press, BMJ Group; 2010.

Next question

Related Articles (BMJ)

10

11

12

13

14

15

16

17

18

19

20

21

22

23

24

25

26

27

28

29

30

31

32

33

34

35

36

37

38

39

40

41

42

Go to summary

43

Revision Notes
Any revision notes recorded
here will be automatically
added to a list, and can be

My Profile

Exam Revision
Work Smart

Scores

Work Hard

Community
Mock Tests

Sign Out

Help
Group Learning 5

Revision Advice

Tags

Learning Journal

Work Smart
Mr VU is taking amitriptyline for depressive illness.

Test Analysis

He comes to see you at a routine outpatient appointment and informs you that he has been experiencing some
side effects with his amitriptyline therapy.
Which of the following is the most commonly recognised side effect of this drug?
(Please select 1 option)
Gout
Hypokalaemia
Renal stones
Taste disturbances
Urinary retention

Score:
Submit answer

Skip question

Go to summary

Total Answered:

Question Navigator
1

10

11

12

13

14

15

16

17

18

19

20

21

22

23

24

25

26

27

28

29

30

31

32

33

34

35

36

37

38

39

40

41

42

43

Tags
Tag
Tag

Hide Question

2014 BMJ Publishing Group Ltd . All rights reserved.


Contact . Corporate . Terms and conditions & Privacy policy

My Profile

Exam Revision
Work Smart

Scores

Work Hard

Community
Mock Tests

Sign Out

Help
Group Learning 5

Revision Advice

Tags

Learning Journal

Work Smart
Mr VU is taking amitriptyline for depressive illness.
He comes to see you at a routine outpatient appointment and informs you that he has been experiencing some
side effects with his amitriptyline therapy.

Answer Statistics
Test Analysis

Which of the following is the most commonly recognised side effect of this drug?
(Please select 1 option)
Gout

Incorrect answer selected

Hypokalaemia
Renal stones
Taste disturbances
Urinary retention

This is the correct answer

Many adverse effects of amitriptyline and similar tricyclic antidepressants are caused by their antimuscarinic
actions.
Antimuscarinic effects are relatively common and occur before an antidepressant effect is obtained. They
include:
Dry mouth
Constipation occasionally leading to paralytic ileus
Urinary retention
Blurred vision and disturbances in accommodation
Increased intraocular pressure and
Hyperthermia.
Tolerance is often achieved if treatment is continued and adverse effects may be less troublesome if treatment
is begun with small doses and then increased gradually, although this may delay the clinical response.

Score:
Total Answered:

Feedback
Rate this question
Leave question feedback

Question Navigator

Drowsiness may also be common, although a few tricyclic antidepressants possess little or no sedative
potential and may produce nervousness and insomnia.

10

11

12

Other neurological adverse effects include:

13

14

15

16

17

18

19

20

21

22

23

24

25

26

27

28

29

30

31

32

33

34

35

36

37

38

39

40

41

42

Headache
Peripheral neuropathy
Tremor
Ataxia
Epileptiform seizures
Tinnitus and
Occasional extrapyramidal symptoms including speech difficulties (dysarthria).

43

Confusion, hallucinations, or delirium may occur, particularly in the elderly, and mania or hypomania, and
behavioural disturbances (particularly in children) have been reported.

Revision Notes

Gastrointestinal complaints include:

Any revision notes recorded


here will be automatically

Sour or metallic taste

added to a list, and can be

Stomatitis and
Gastric irritation with nausea and vomiting.

viewed in full on the summary


page.

Orthostatic hypotension and tachycardia can occur in patients without a history of cardiovascular disease, and
may be particularly troublesome in the elderly.
Hypersensitivity reactions, such as urticaria and angioedema, and photosensitisation have been reported and,
rarely, cholestatic jaundice and blood disorders, including:
Eosinophilia
Bone marrow depression
Thrombocytopenia
Leucopenia and
Agranulocytosis.

Save

Endocrine effects include testicular enlargement, gynaecomastia and breast enlargement, and galactorrhoea.
Sexual dysfunction may also occur.
Changes in blood sugar concentrations may also occur, and, very occasionally, hyponatraemia associated with

Tags
Tag
Tag

inappropriate secretion of antidiuretic hormone.

Hide Question
Other adverse effects that have been reported are increased appetite with weight gain (or occasionally
anorexia with weight loss). Sweating may be a problem.
Martindale
BNF 60

Next question

Go to summary

Related Articles (BMJ)

2014 BMJ Publishing Group Ltd . All rights reserved.


Contact . Corporate . Terms and conditions & Privacy policy

My Profile

Exam Revision
Work Smart

Scores

Work Hard

Community
Mock Tests

Sign Out

Help
Group Learning 5

Revision Advice

Tags

Learning Journal

Work Smart
A patient is prescribed warfarin for prophylaxis of DVT.

Test Analysis

Which vitamin does warfarin antagonise?


(Please select 1 option)
A
B6
C
D
K

Submit answer

Skip question

Go to summary

Score:
Total Answered:

Question Navigator
1

10

11

12

13

14

15

16

17

18

19

20

21

22

23

24

25

26

27

28

29

30

31

32

33

34

35

36

37

38

39

40

41

42

43

Tags
Tag
Tag

Hide Question

2014 BMJ Publishing Group Ltd . All rights reserved.


Contact . Corporate . Terms and conditions & Privacy policy

My Profile

Exam Revision
Work Smart

Scores

Work Hard

Community
Mock Tests

Sign Out

Help
Group Learning 5

Revision Advice

Tags

Learning Journal

Work Smart
A patient is prescribed warfarin for prophylaxis of DVT.

Answer Statistics

Which vitamin does warfarin antagonise?

Test Analysis

(Please select 1 option)


A

Incorrect answer selected

B6
C
D
K

This is the correct answer

Warfarin inhibits hepatic vitamin K epoxide reductase, which is an enzyme that converts vitamin K to its active
form (hydroquinone).
This results in the impairment of the hepatic synthesis of vitamin K dependent clotting factors (II [prothrombin],
VII, IX, and X).
Score:
Next question

Go to summary

Total Answered:

Feedback
Related Articles (BMJ)

Rate this question


Leave question feedback

Question Navigator
1

10

11

12

13

14

15

16

17

18

19

20

21

22

23

24

25

26

27

28

29

30

31

32

33

34

35

36

37

38

39

40

41

42

43

Revision Notes
Any revision notes recorded
here will be automatically
added to a list, and can be

My Profile

Exam Revision
Work Smart

Scores

Work Hard

Community
Mock Tests

Sign Out

Help
Group Learning 5

Revision Advice

Tags

Learning Journal

Work Smart
A patient on your ward is prescribed warfarin as she has recently been diagnosed with atrial fibrillation. Her

Test Analysis

desired INR is 2.5.


On the morning ward round you take the patient's INR which comes back as 5.2 from the laboratory. There are
no signs of bleeding.
What would be your next course of action?
(Please select 1 option)
Decrease the dose of warfarin
Do nothing, as there are no signs of bleeding
Increase the dose of warfarin
Omit the warfarin
Start a heparin infusion

Score:
Submit answer

Skip question

Go to summary

Total Answered:

Question Navigator
1

10

11

12

13

14

15

16

17

18

19

20

21

22

23

24

25

26

27

28

29

30

31

32

33

34

35

36

37

38

39

40

41

42

43

Tags
Tag
Tag

Hide Question

2014 BMJ Publishing Group Ltd . All rights reserved.


Contact . Corporate . Terms and conditions & Privacy policy

My Profile

Exam Revision
Work Smart

Scores

Work Hard

Community
Mock Tests

Sign Out

Help
Group Learning 5

Revision Advice

Tags

Learning Journal

Work Smart
A patient on your ward is prescribed warfarin as she has recently been diagnosed with atrial fibrillation. Her

Answer Statistics

desired INR is 2.5.


On the morning ward round you take the patient's INR which comes back as 5.2 from the laboratory. There are
no signs of bleeding.

Test Analysis

What would be your next course of action?


(Please select 1 option)
Decrease the dose of warfarin

Incorrect answer selected

Do nothing, as there are no signs of bleeding


Increase the dose of warfarin
Omit the warfarin

This is the correct answer

Start a heparin infusion

The main adverse effect of all oral anticoagulants is haemorrhage.


Checking the INR and omitting doses when appropriate is essential; if the anticoagulant is stopped but not
reversed, the INR should be measured two to three days later to ensure that it is falling.
The following recommendations are based on the result of the INR and whether there is major or minor
bleeding; the recommendations apply to patients taking warfarin:
Major bleeding - omit warfarin; give phytomenadione (vitamin K1) 5-10 mg by slow intravenous injection;
give dried prothrombin complex (factors II, VII, IX, and X) 30-50 units/kg (if dried prothrombin complex
unavailable, fresh frozen plasma 15 mL/kg can be given but is less effective).
INR 8.0, no bleeding or minor bleeding - omit warfarin and give phytomenadione (vitamin K1) 2.5-5 mg by
mouth using the intravenous preparation orally [unlicenced use], or 0.5-1 mg by slow intravenous injection
(if complete reversal required 5-10 mg by slow intravenous injection); repeat dose of phytomenadione if
INR still too high after 24 hours; restart warfarin when INR less than 5.0.
INR 5.0-8.0, no bleeding - omit warfarin; minor bleeding - omit warfarin and give phytomenadione (vitamin
K1) 1-2.5 mg by mouth using the intravenous preparation orally [unlicenced use]; restart warfarin when INR
less than 5.0.
Unexpected bleeding at therapeutic levels - always investigate possibility of underlying cause, for example,
unsuspected renal or gastrointestinal tract pathology.

Next question

Go to summary

Score:
Total Answered:

Feedback
Rate this question
Leave question feedback

Question Navigator
1

10

11

12

13

14

15

16

17

18

19

20

21

22

23

24

25

26

27

28

29

30

31

32

33

34

35

36

37

38

39

40

41

42

43

Related Articles (BMJ)


Revision Notes
Any revision notes recorded
here will be automatically
added to a list, and can be

My Profile

Exam Revision
Work Smart

Scores

Work Hard

Community
Mock Tests

Sign Out

Help
Group Learning 5

Revision Advice

Tags

Learning Journal

Work Smart
Mr YB is a patient who regularly attends the anticoagulant clinic.

Test Analysis

He is very concerned as he has been recently started on a new drug by his GP. He asks you whether it would
enhance the anticoagulant effect.
Which of the following may increase the potential for bleeding in patients taking warfarin?
(Please select 1 option)
Clopidogrel
Carbamazepine
Griseofulvin
Phenobarbitone
St. John's wort

Score:
Submit answer

Skip question

Go to summary

Total Answered:

Question Navigator
1

10

11

12

13

14

15

16

17

18

19

20

21

22

23

24

25

26

27

28

29

30

31

32

33

34

35

36

37

38

39

40

41

42

43

Tags
Tag
Tag

Hide Question

2014 BMJ Publishing Group Ltd . All rights reserved.


Contact . Corporate . Terms and conditions & Privacy policy

My Profile

Exam Revision
Work Smart

Scores

Work Hard

Community
Mock Tests

Sign Out

Help
Group Learning 5

Revision Advice

Tags

Learning Journal

Work Smart
Mr YB is a patient who regularly attends the anticoagulant clinic.

Answer Statistics

He is very concerned as he has been recently started on a new drug by his GP. He asks you whether it would
enhance the anticoagulant effect.

Test Analysis

Which of the following may increase the potential for bleeding in patients taking warfarin?
(Please select 1 option)
Clopidogrel

Correct

Carbamazepine
Griseofulvin
Phenobarbitone
St. John's wort

Clopidogrel does not appear to have a clinically relevant effect on the pharmacokinetics or pharmacodynamics
of warfarin.
Score:

However, the concurrent use of clopidogrel with warfarin, increases the bleeding risk.
All other drugs in the options are C-P450 enzyme inducers so would decrease the anticoagulant effect
Reference:

Feedback

Baxter K, editor. Stockley's Drug Interactions. 9th ed. London: Pharmaceutical Press; 2011.

Next question

Total Answered:

Rate this question

Go to summary

Leave question feedback

Question Navigator
Related Articles (BMJ)

10

11

12

13

14

15

16

17

18

19

20

21

22

23

24

25

26

27

28

29

30

31

32

33

34

35

36

37

38

39

40

41

42

43

Revision Notes
Any revision notes recorded
here will be automatically
added to a list, and can be

My Profile

Exam Revision
Work Smart

Scores

Work Hard

Community
Mock Tests

Sign Out

Help
Group Learning 5

Revision Advice

Tags

Learning Journal

Work Smart
Which of the following does not have a role in the management of chronic cancer pain?

Test Analysis

(Please select 1 option)


Carbamazepine
Clodronate
Dexamethasone
Nifedipine
Pinaverium

Submit answer

Skip question

Go to summary

Score:
Total Answered:

Question Navigator
1

10

11

12

13

14

15

16

17

18

19

20

21

22

23

24

25

26

27

28

29

30

31

32

33

34

35

36

37

38

39

40

41

42

43

Tags
Tag
Tag

Hide Question

2014 BMJ Publishing Group Ltd . All rights reserved.


Contact . Corporate . Terms and conditions & Privacy policy

My Profile

Exam Revision
Work Smart

Scores

Work Hard

Community
Mock Tests

Sign Out

Help
Group Learning 5

Revision Advice

Tags

Learning Journal

Work Smart
Which of the following does not have a role in the management of chronic cancer pain?

Answer Statistics

(Please select 1 option)


Carbamazepine

Incorrect answer selected

Test Analysis

Clodronate
Dexamethasone
Nifedipine
Pinaverium

This is the correct answer

Pinaverium is used to reduce the pain duration in irritable bowel syndrome (IBS).
Carbamazepine is in use for the treatment of neuropathic pain of malignancy, diabetes and other disorders.
Clodronate inhibits osteoclastic bone resorption and is used to treat malignant bone pain and the associated
hypercalcaemia.
The corticosteroids are used to treat pain from central nervous system tumours. Reducing the inflammation
and oedema relieves the pain caused by neural compression.

Score:
Total Answered:

Nifedipine helps relieve painful oesophageal spasm and tenesmus associated with gastrointestinal tumours.
Painful bladder spasm may be relieved by oxybutynin.

Feedback
Rate this question

Go to summary

Leave question feedback

Related Articles (BMJ)

Question Navigator
1

10

11

12

13

14

15

16

17

18

19

20

21

22

23

24

25

26

27

28

29

30

31

32

33

34

35

36

37

38

39

40

41

42

43

Revision Notes
Any revision notes recorded
here will be automatically
added to a list, and can be

Anda mungkin juga menyukai